April v1.1 2022

You might also like

Download as pdf or txt
Download as pdf or txt
You are on page 1of 188

111

GOLDEN

TEAM

April

1-Patient presented with neck swelling, she is completely asymptomatic, neck US

was done reveal a solid mass of 7 - mm with regular margins Which of the

following is the most appropriate next step?

A. FNA

B. Thyroid scintigraphy

C. Check TSH

• Correct Answer:C

American Thyroid Association: Serum thyrotropin (TSH) should be measured

during the initial evaluation of a patient with a thyroid nodule.

2-Bacterial vaginosis scenario with fishy smell no itching What cells will you see?
A. Granular epithelial cells
B. Atypical cells
C. Budding yeast

• Correct Answer:A

AMBOSS: Diagnosis is confirmed if three of the following Amsel criteria are met [3]

• Clue cells (vaginal epithelial cells with a stippled appearance and fuzzy borders
(granular) due to bacteria adhering to the cell surface
• Vaginal pH > 4.5
• Positive Whiff test
• Thin, homogeneous gray-white or yellow discharge that adheres to the vaginal
walls

3-Which of the following nutrients fail to get absorbed in a patient with cystic
fibrosis?
A. Vitamins A, D, E, and B complex
B. Vitamins A, B, E, and K
C. Vitamins B1, B2, B12
D. Vitamins A, D, E, and C
E. Vitamins A, D, E, and K

• Correct Answer:E

AMBOSS: Deficiency of fat-soluble vitamins (KADE)

2
4-Female complaining of abnormal uterine bleeding she has an endometrial polyp,
on US endometrial lining was 19mm, what will you offer to this patient at this
stage?
A) open hysterectomy
B) laparoscopic hysterectomy
C) hysteroscopy with polypectomy

• Correct Answer:C

AMBOSS: Asymptomatic women: observation and follow-up, Symptomatic women:


surgical removal (via hysteroscopy)

5- 82-year-old male with very painful micturition what is the most appropriate
management?
A. Abx for uti
B. Foley catheter
C. Cysoscopy and turp

• Correct Answer:A

PubMed: UTI is the most common indication for antibiotic prescriptions in older
adults.
Note: ‫ بس اغلب الي اختبروا يقولون صيغه السؤال كانت غير وكان عنده‬retention which
indicates BPH > Foley catheter

6-The MOH is organizing campaigns, lectures, and health education in order to


teach the public about the dangers of obesity and its associated complications.
What kind of prevention is this?
A. Tertiary
B. Secondary
C. Primary
D. Primordial

3
• Correct Answer: D

PubMed: Primordial prevention It consists of risk factor reduction targeted


towards an entire population through a focus on social and environmental
conditions.
Primary prevention aims to prevent disease or injury before it ever occurs.
Secondary prevention aims to reduce the impact of a disease or injury that has
already occurred. (e.g. screening programs)
Tertiary prevention aims to soften the impact of an ongoing illness or injury that
has lasting effects. (e.g. rehabilitation programs)

7- The government has decided to set up campaigns to help increase the


awareness of hypertension to the public, as well as educate them about the risk
factors of hypertension and encourage low salt diets. What kind of prevention is
this?
a. Primordial
b. Primary
c. Secondary
d. Tertiary

• Correct Answer:A (Multiple campaigns organized by the government +


education about the risks not the disease itself).

8- Secondary prevention of diabetes is:


A-Screening
B- chronic management of disease
C- education in health and disease

• Correct Answer:A

9-Pregnant woman has Hepatitis B which type of prevention we should preformed:

4
‫‪A- primary‬‬
‫‪B- secondary‬‬
‫‪C- tertiary‬‬

‫‪• Correct Answer:C‬‬

‫‪Note:‬‬
‫‪primordial‬‬
‫‪ :‬باختصار اذا شوفتوا السؤال يقول‬
‫ساسية صحية = ‪Policy‬أو رضائب = ‪Taxes‬‬

‫‪ :‬اذا شوفتوا ‪ Primary‬بينما ال‬


‫‪Health Education‬‬
‫‪Health promotion‬‬
‫حمالت توعية =‪Awareness‬‬

‫‪ Secondary :‬ال‬
‫للمرض ‪ Screening‬مع ‪ S‬يكون من اسمها حرف‬

‫‪Primordial‬‬
‫التدخي ر يف االماكن العامه‪ ،‬ممنوع استخدام الزيوت‬
‫ر‬ ‫ش يتعلق بالحكومات والوزارات‪ .‬مثال قرار يطلع ممنوع‬
‫ي‬
‫ال‬ ‫االطفال‬ ‫العاب‬ ‫بيع‬ ‫ممنوع‬ ‫‪،‬‬ ‫ر‬
‫التدخي‬ ‫ائب‬ ‫ض‬ ‫المهدرجه‪ ،‬يجب وضع السعرات الحراريه لكل وجبه‪ ،‬زيادة ر‬
‫ي‬
‫تكون‬
‫شكل دخان ‪،‬‬
‫توع الناس تكون‬
‫‪ Primary‬ولكن اذا الوزاره طلعت تسوي حمله هدفها ي‬

‫‪5‬‬
6
10-Patient received 5mg warfarin his inr is 7 , known case of AF ?
A. Reduce to 2.5 mg
B. Same dose
C. Stop warfarin
D. hold and repeat inr

• Correct Answer:D

UpToDate: the INR is between 4.5 and 10 and the patient does not have clinically
significant bleeding, warfarin is held temporarily (eg, one or two doses), with or
without administration of a small dose of oral vitamin K. The INR should be
monitored more frequently during the period of observation
and warfarin reinitiation (eg, every 24 to 72 hours until warfarin is resumed).

11- breast feeding mother found unilateral mass on the lower outer quadrant of
right breast (i think 3x3 cm) it was erythematous and hot on palpation. Mildly
tender. Most appropriate management?
A. chemo therapy
B. radiation therapy
C. needle core biopsy
D. Aspiration + Abx

• Correct Answer:D

7
AMBOSS: Treatment of breast abscess (complication of mastitis)
• Needle aspiration: for abscesses with intact overlying skin
• Incision and drainage: if overlying skin necrosis is present
• Antibiotic treatment

12- Which of the following dermatological condition is associated with Sarcoidosis?


A. Erythema toxicum
B. Erythema nodosum
C. Erythema multiforme
D. Erythema larva migrans
E. Erythema marginatum

• Correct Answer:B

AMBOSS: Extrapulmonary symptoms: arthritis, anterior uveitis, erythema nodosum

13-Which of the following findings does not indicate a poor prognosis for
pancreatitis?
A. Metabolic acidosis
B. Hypocalcemia
C. Coagulopathy
D. Hypoglycemia
E. Hyperglycemia

• Correct Answer:D

AMBOSS: Important predictors of severity


• Age > 55
• Gastrointestinal bleeding
• Fall in Hct within 48 hours
• Hypocalcemia and/or hyperglycemia
• Inflammatory markers: ↑↑ CRP, ↑ IL-6, ↑ IL-8
• Evidence of shock and/or organ failure
o ↑ AST, ↑ ALT

8
o ↑ BUN, creatinine
o ↑ LDH
o ABG: pO2 < 60 mm Hg, metabolic acidosis with a base deficit > 4 mmol/L

• CT findings: pancreatic edema, peripancreatic fluid collection,


and/or necrosis of > 33% of the pancreas

14- Which of the following is not a sign of severe immediately life-threatening


asthma?
A. Reduced level of consciousness
B. Bradycardia
C. Inaudible breath sounds on lung auscultation
D. Reduced PaCO2

• Correct Answer:D

UpToDate: Respiratory drive is almost invariably increased in acute asthma,


resulting in hyperventilation and a correspondingly decreased PaCO2. Thus, a
normal PaCO2 (eucapnia) during an asthma exacerbation indicates that airway
narrowing and dynamic hyperinflation are so severe that tidal volume and alveolar
ventilation are starting to decrease, despite persistent intense central respiratory
drive. Hypercapnia and respiratory failure can then develop rapidly with any further
airway obstruction or with respiratory muscle fatigue.

15- Woman did some procedure (pelvic procedure but forgot what it was) and
then came complaining of urine from vagina during urination (micturition) What is
the dx?
A.Vesicovaginal fistula
B. Ureterovaginal fistula
C. Urethrovaginal fistula
D.Rectovaginal fistula

9
• Correct Answer:C

Because during micturation

16- A child starts to develop an awareness for strangers and separation anxiety.
How old is the child in months?
A. 6 months
B. 7 months
C. 12 months
D. 24 months

• Correct Answer:C

Stranger anxiety at 6 months


Separation at age of 12

10
17-Patient 60 something k/c of dyslipidemia on statin report high blood pressure
reading of
150/90 In the clinic Labs all normal, Bp: 145/90 What is your management?
A- Start amlodipine.
B- life style modification and weight reduction.

• Correct Answer:A

UpToDate: The decision to initiate drug therapy should be individualized. In


general, we suggest that antihypertensive drug therapy be initiated in the
following:
Patients with out-of-office daytime blood pressure ≥135 mmHg systolic or ≥85
mmHg diastolic (or an average office blood pressure ≥140 mmHg systolic or ≥90
mmHg diastolic if out-of-office readings are not available).

18- Pancreatitis 5 weeks ago. Now she has epigastric tenderness and cannot
tolerate food with vomiting each time. By ultrasound you found large about 12X10
mass with thick wall and fluid inside. Labs: 346 amylase, Wbc 15k. What is the
diagnosis?
A. Pseudocyst
B. Abscess
C. Walled off necrosis

• Correct Answer:A

AMBOSS: Encapsulated collection of pancreatic fluid that develops 4 weeks after


an acute attack of pancreatitis. Extrapancreatic fluid collection within well-defined
wall or capsule.

11
19-A female get high energy accident (high velocity), with seat belt sign, on X ray
have chance fracture. What will associated with this fracture?
A- Duodenal perforation
B- Gastric perforation
C- Jejunum perforation
D- Vena cava perforation

• Correct Answer: A

20-15-year-old girl has sparse pubic hair not reaching the mons pubis, breast
budding with areolar enlargement, but with no clear distinction between breast
and areola. No contour of breast was seen. What Tanner stage is she?
a. Tanner I
b. Tanner II
c. Tanner III
d. Tanner IV

• Correct Answer:B

12
21- G2p0 20w gestation with cervical incompetence (cervix length 30),
Management??
A-cervical cerclage
B-progesterone supplementation
C-strict bed rest

• Correct Answer:B

UpToDate: We suggest cerclage placement in patients with one prior spontaneous


preterm birth and TVU CL ≤25 mm before 24 weeks in the current pregnancy.
Another alternative approach is to treat these patients with
vaginal progesterone alone, without cerclage placement. The author does not
advise this approach.

Note:
Depends if 30 mm and cervix open not contracted =A
30 mm and cervix closed=B

22- What anti diabetic medication can reduce mortality in DM patients?


A. Metformin
B. Gliflozin
C. Glipizide
D. Acarbose

• Correct Answer:B

PubMed: Recently several medications that reduce HbA1c such as the sodium
glucose co-transporter (SGLT2) inhibitors, empagliflozin (EMPA REG outcomes trial)

13
and glucagon-like peptide (GLP)-1 agonists, liraglutide and semaglutide have
improved mortality and cardiovascular risk factors in type 2 diabetic patients.

23- Hereditary spherocytosis picture, what will you order?

• Correct Answer:Osmotic fragility test

24- Baby with a rash, both parents have eczema, where do you think the location
of the rash will be?
Correct Answer:On face and scalp.
Infants younger than 1 year old usually have the eczema rash on their cheeks,
forehead, or scalp.

25-APGAR Score Q:

26-Patient who’s diabetic, develops liver symptoms, dx?


A-Nonfatty alcoholic jaundice
B-viral jaundice

14
• Correct Answer:A

Pubmed: T2DM and NAFLD have a common association. Diabetes promotes the
progression of NAFL to NASH and increases the risk of cirrhosis and HCC.

27- Post splenectomy when to give vaccine?


A-2 weeks
B-2 months
C-6 months

• Correct Answer:A

CDC

28- Patient with asthma symptoms that are not controlled although he uses
Bronchodilators, corticosteroids, & LABA. His PFTs are 75% before the use of a
bronchodilator and 95% after using it. What to do?

• Correct Answer:Observe how he uses the inhaler

29- Screening for asymptomatic bacteriuria in pregnancy:


a) 12 weeks
b) 20 weeks
c) 27 weeks
d) 31 or 32 weeks

• Correct Answer:A

The United States Preventive Services Task Force recommends screening for
asymptomatic bacteriuria with urine culture for pregnant women at 12 to 16
weeks’ gestation or at their first prenatal visit, if later.

15
30- Child came to clinic and the doctor advised his parents to start iron
supplements next month, how old is the child now?

• Correct Answer:3 months

PubMed: Start a source of iron (supplemental iron at 1 mg/kg/day or iron-


fortified infant cereal or meat) at age 4 months.

31- Child who plays in the farm or garden something like that, he has increased
saliva, tearing, diarrhea, sweating.. dx?

• Correct Answer:Organophosphate toxicity

32- Child with hip joint pain and refuses touching it, what is the organism?

• Correct Answer:S.aureus

AMBOSS: Staphylococcus aureus, most common in adults and children > 2 years.

33- A patient with a prosthetic valve who is about to do a dental procedure, which
prophylactic antibiotics?
A-Amoxicillin
B-Penicillin

• Correct Answer:A

UpToDate: For patients with relevant cardiac risk factors undergoing dental
procedures, the preferred regimen is oral amoxicillin.

16
34- Pt 10 years old no co-morbidities with history of rheumatic fever with no
cardiac involvement how long should he receive prophylactic antibiotics for?
A-5 years
B-10 years

• Correct Answer:B

UpToDate: 5 years or until 21 whichever is longer

35- Which of the following indicates compensated shock?


A) Anuria
B) Confusion
C) Hypotension
D) cold and pale peripheries

• Correct Answer:D

17
36- What decreases mortality rate in esophageal varices?

• Correct Answer:IV ceftriaxone

UpToDate: For patients with cirrhosis and gastrointestinal (GI) bleeding who are
hospitalized, prophylactic antibiotics reduce the risk of mortality, infections (eg,
spontaneous bacterial peritonitis, urinary tract infections), and rebleeding. We
typically use a broad-spectrum antibiotic such as ceftriaxone.

18
37- Female has molar pregnecy B-hCG at begining was (856 000)
On week one (560)
Week 2 (360)
Week 3 (160)
Week 4 (56)
Week 5 1. Yesitisone
*numbers are not exactly same but somehow near
How you will follow the patient?
A. Measurement at weekly interval
B. Measure at one-month interval
C. Discharge from clinin (sure was like this)
D. Repeat ultrasound

• Correct Answer:B

UpToDate: Therefore, in patients with complete mole, after one normal hCG, we
obtain monthly hCG values for three additional months and then discontinue
monitoring if the level remains undetectable. In patients with partial mole, after
hCG normalizes, we obtain a single additional hCG measurement one month later
and then discontinue monitoring if the level remains undetectable.

DR.Wafa says the answer is (A), - The initial B-hCG level is obtained within 48 hours
after evacuation. (baseline) - Then weekly until undetectable - Once B-hCG is
undetectable(for three consecutive weeks), this is confirmed with monthly for
another 6 months

38- Pregnant bp 140/90 or 150/90 what to give ? “Exactly like this”


A- nifidepine
B- hydralazine
C- methyldopa
No labetalol in choices

• Correct Answer:A

UpToDate: We prefer labetalol or intermediate-acting or extended-


release nifedipine. Oral hydralazine may be added if needed to achieve and
maintain target blood pressure.

19
39- 50 yr female c/o menorrhagia for one year, she has fibroid 7x7 medical tx
failed, what’s the most appropriate for her?
A- hysterectomy
B- uterine artery embolization
C- Hormonal IUD

• Correct Answer:A

UpToDate: For patients who do not desire future fertility and have persistent
fibroid-related symptoms despite the above therapies or who desire surgical
treatment, options include hysterectomy and myomectomy. Hysterectomy is
reserved for patients who previously have had unsuccessful conservative therapy
or who have significant concomitant diseases.

40- Female in 30s or 40s has excised fibroadenoma, histopathology showed (forgot
type of cells) with hyperplasia and atypia, which factor suggests malignancy?
A-Age
B-Presence of Atypia
C-Presence of hyperplasia

• Correct Answer:B

UpToDate: Proliferative lesions with atypia are considered high risk because they
are associated with an increase in the patient's future risk of developing breast
cancer.

41- During the examination the doctor can feel the presenting part is soft?
A- face
B- Brow
C⁃ Breech

• Correct Answer:C

UpToDate: Breech presentation is characterized by the presence of a soft mass (ie,


buttocks) and the absence of a hard fetal skull.

20
42- Patient with SLE symptoms what's the most confirmatory test?
A-ANA
B- Anti DsDNA

• Correct Answer:B

AMBOSS: Anti-dsDNA antibodies is highly specific for SLE.

43- Case SLE + very high ferritin very low hbg high alkaline phosphatase? Most
likely anemia?
A-Anemia of chronic dx
B-Hemolytic
C-Pernicious
D-Aplastic

• Correct Answer:A

UpToDate: In anemia of chronic disease, transferrin is generally decreased and


ferritin is generally increased.

44- Case of rheumatoid arthritis compliance on MTx, improve and stable for years
without attack, the LFT abnormal, what will you do?
A- stop MTx
B- adalimumab

• Correct Answer:A

UpToDate: MTX should be discontinued if no other reasons for transaminase


enzymes elevations are found and the enzymes remain elevated above the
laboratory-defined upper limit of normal.

21
45- At which age can we give peanuts and eggs to babies to prevent the risk of
getting allergic?
A. 10 months
B. 14 months
C. 18 months
D. 24 months

• Correct Answer:A

UpToDate: Several studies have not supported delaying the introduction of solid
foods beyond four to six months for the prevention of allergic disease.
Australasian Society of Clinical Immunology and Allergy: It is best to offer your
baby well cooked egg and smooth peanut butter/paste starting before 12 months
of age. Delayed introduction of these foods has been shown to increase the chance
of developing food allergy.

46- 27 y old female present with acute lower abdominal pain radiate to left
shoulder what is the highest diagnostic investigation?
A. Pelvic CT
B. Abdominal MRI
C. Pregnancy test

• Correct Answer:C

AMBOSS: Every woman of reproductive age with abdominal pain should undergo a
pregnancy test, regardless of contraception use.

48- Scenario with symptoms of hypocalcemia (mouth twitching and spasm), what’s
Tx:
A. Oral calcium
B. IV calcium

• Correct Answer:B

22
AMBOSS: Severe and/or symptomatic
hypocalcemia: e.g., tetany, seizures, prolonged QT interval, serum calcium ≤ 7.5
mg/dL, give IV calcium supplementation.

49- Strawberry cervix and yellow green discharge?

• Correct Answer:Trichomonas vaginitis

AMBOSS: Clinical features: Foul-smelling, frothy, yellow-green, purulent discharge,


vulvovaginal pruritus, burning sensation, dyspareunia, dysuria, strawberry
cervix (erythematous mucosa with petechiae)

50-A baby with stridor noisy breathing and wheezing that improves when prone
and increases when supine. What is the diagnosis?
A.chest X-ray
B. Nasopharyngeal something
C. Respiratory culture
D.improve with his first birthday

• Correct Answer:D

AMBOSS: In Laryngomalacia the inspiratory stridor is usually worse when the child
is in a supine position, when crying or agitated, or when an upper respiratory tract
infection occurs. Treatment is reassurance (resolves by age 2 years in 90% of
cases).

51- 5 y/o boy one testicles undescended. Mx?


A-Orchidectomy
B-Orchidopexy
C-Wait till puberty

• Correct Answer:B

UpToDate: Orchidopexy is recommended as soon as possible after four months of


age for congenitally undescended testes and definitely should be completed
before the child is two years old (ideally before one year). In children with

23
testicular ascent later in childhood, surgery generally should be performed within
six months of identification.
AMBOSS: Orchiectomy: in cases of nonviable testicular remnants or late discovery
of undescended testicle (> 2 years)

52- Pt Elderly tired, thirsty on ex abdomen tender rigid and for investigation What
is next in management?
A-Hydration
B-Endoscopy
C-Antibiotics

• Correct Answer:A

Start with the ABC’s.

53- Trauma patient. Vitally was stable. FAST done was positive next step?
A-Peritoneal lavage
B-Ct abdomen
C-Laparotomy
D-Diagnostic laparoscopy

• Correct Answer:B

UpToDate: CT is highly sensitive for both intraperitoneal hemorrhage and solid


organ injury and is the imaging study of choice in stable patients with suspected
intraabdominal or intrathoracic injury.

54- child with perforated drum and discharge come out what is the diagnosis?
A-otitis media
B-otitis externa

• Correct Answer:A

UpToDate: serous otitis media is defined as the presence of middle ear fluid
without signs of acute infection. Perforation happens as a complication, the
increased pressure in the middle ear can result in central ischemia, necrosis, and
spontaneous perforation of the TM, usually accompanied by otorrhea.

24
55- Treatment of brucellosis (I am sure no neurological symptom):
A-6 weeks
B-3 weeks
C-3 months
D-6 months

• Correct Answer:A

UpToDate: Doxycycline (oral) for 6 weeks PLUS streptomycin (parenteral) for the
first 14 to 21 days.

56- 12 years old found to have klebsiella colony 100000 from mid urine no sign or
symptom:
A-no need to treat
B-treat as acute UTI
C-give empirical antibiotic

• Correct Answer:A

PubMed: The current recommendation is not to treat asymptomatic bacteriuria in


the pediatric population, except for renal transplant recipients and children
undergoing urologic procedures.

57- Type of Vit D in child with rickets?


A-D1
B-D2
C-D3

• Correct Answer:C

PubMed: Some studies suggest that cholecalciferol (Vitamin D3) increases serum
25(OH) D more efficiently than does ergocalciferol (Vitamin D2).

58- 12 years old boy diagnosed with DM type 1 he has been complaining with
hypoglycemia since he takes his medication 2 month ago and not reach control?
A-brittile phenomena

25
B-down phenomena
C-somogyi phenomena
D-honeymoon phenomena

• Correct Answer:D

UpToDate: A few weeks after the diagnosis and initiation of insulin therapy, a
period of decreasing exogenous insulin requirement occurs, commonly referred to
as the "honeymoon" or remission phase of diabetes. During this period, the
remaining functional beta cells secrete some endogenous insulin, resulting in
reduced exogenous requirement. Close monitoring of blood glucose is mandatory
since hypoglycemic episodes are likely if the insulin dose is not appropriately
adjusted. Rising blood glucose levels, A1C, and increasing exogenous insulin need
indicate the end of this phase.

59- Pediatric known case of HF and HTN, lower limb edema and dyspnea on
exertion what to give?
A-frusemide
B-reassure
C-Beta Blocker

• Correct Answer:A

UpToDate: Diuretics decrease preload by promoting natriuresis and provide relief


of volume overload symptoms such as pulmonary and peripheral edema.
Furosemide is the most used loop diuretic.

60- child came with fever cough and diffuse wheeze what is the management?
A-admit for O2 and antibiotics (not sure)
B-admit for O2 and fluid
C-discharge patient with antibiotic

• Correct Answer:B

UpToDate: The need to monitor fluid intake and output; children with bronchiolitis
may have difficulty maintaining adequate hydration because of increased needs

26
(related to fever and tachypnea) and decreased intake (related to tachypnea and
respiratory distress). Supplemental oxygen is provided as necessary to maintain
SpO2 >90 to 92 percent. Antibiotics are indicated only if there is evidence of a
coexisting bacterial infection.

61- 38 weeks delivery complain from tachypnea and granting what is the
diagnosis?
A-transient tachypnea attack
B-respiratory distress syndrome
C-meconium aspiration

• Correct Answer:A

AMBOSS: choice A-Most commonly in full-term and near-term infants


B-In preterm, C-in postterm

62- child with foreign body aspiration where is the exact location?
A-rt bronchi
B-left bronchi

• Correct Answer:A

AMBOSS: the right main bronchus is more often affected than the left
main bronchus. The right main bronchus is orientated more vertically than the left
main bronchus, as the heart is in the left lower thorax.

63- child with inhaled seeds for several month and his mother bring him many
times to hospital and discharge him now came with wheeze and pneumonia in
right lower lung how to treat?
A. rigid bronchoscope
B. flexible bronchoscope

• Correct Answer:B

UpToDate: If FBA is known to have occurred, rigid bronchoscopy is the procedure


of choice to identify and remove the object. If there is a suspicion for multiple
small FBs or fragments, we recommend performing a complete flexible
bronchoscopy after FB removal to evaluate the entire tracheobronchial tree. If the

27
FB has been retained long enough for an infection to occur, a Gram stain and
culture should be obtained through the bronchoscope. Occasionally, an FB that has
been retained for several weeks will cause such intense airway inflammation and
infection that it cannot be removed. In such cases, antibiotics should be
administered, guided by Gram stain and cultures obtained at bronchoscopy. In
addition, a three- to seven-day course of systemic corticosteroids.

64- 9 years old child came with his mother with anuresis what is the muscle
affected

• Correct Answer:detrusor

AMBOSS: The bladder contains smooth muscle (the detrusor muscle of


the bladder) that contracts during micturition.

65- patient came with sign and symptom of epiglotitis drolling saliva and severe ill
ask about treatment?
A-discharge with antibiotic
B-intubate the patient and admit to ICU

• Correct Answer:B

UpToDate: Children older than six years of age and adult patients with epiglottitis
and severe respiratory distress (eg, stridor, drooling, sitting erect, cyanosis) or >50
percent obstruction of the laryngeal lumen by endoscopic evaluation also warrant
endotracheal intubation. Most adults without signs of severe airway obstruction
can be managed in an intensive care.

67- 1m year old came with eczema on the face trunk elbow what is the diagnosis?
A-infantile eczema
B-Idiopathic urticaria

• Correct Answer:A

AMBOSS: Eczema involving the face, head, and extensor surfaces of the
extremities that usually spares the diaper area.

28
68- Pregnant with suspicious of cervix lesion what next?
a. Cone biopsy
b. Curettage
c. Colposcopy
d. Pap

• Correct Answer:C Dr.Wafaa

UpToDate: -Cervical cancer is often first suspected when a screening test (Pap)
for the disease is abnormal.
- If colposcopy is performed, cervical biopsy should be performed only if a lesion is
present that appears to be high-grade.
-Endocervical curettage and endometrial biopsy should not be performed.
-Traditional indications for cervical conization in the nongravid population are not
applicable during pregnancy.
-Cervical biopsies ---> relatively contraindicated
• Pap smear -> indicated as 1st trimester test (routine)
• Colposcopy -> can be done
• Ectocervix biopsy -> can be done
• Endocervical curettage (ECC) -> CONTRAINDICATED in pregnancy

69- 31-year-old female was following up for the last 5 year with women health, last
year negative and this year, negative Pap smear and negative HPV Test, next time
to do Pap smear:
a) 6 months
b) 1year
c) 2 years
d) 3years

• Correct Answer:D

UpToDate: Age ≥30 years, One of these methods:


▪ Pap test every 3 years

▪ Primary HPV testing alone every 5 years
▪ Co-testing (Pap test and HPV testing) every 5 years

29
70- single umbilical artery what you suspect?

• Correct Answer:diabetic mother

UpToDate: SUA has been associated with maternal smoking, diabetes,


hypertension, and seizure disorders.

71- An adnexal mass was felt in healthy female with normal pelvic examination, no
pregnancy, had her menstruation 2 weeks?
a) follicular cysts
b) luteal cysts
c) PCOS

• Correct Answer:A

AMBOSS: Follicular cyst of the ovary (most common ovarian mass in young
women), usually asymptomatic unless complications occur, adnexal mass that is
sometimes palpable.
luteal cysts are common during pregnancy.

Extra Note:
-Cystocele only = anterior colpoperineorrhaphy
-Rectocele only= posterior colpoperineorrhaphy
-Cystorectocele= Ant&post colpoperineorrhaphy
-Both of them + uterine prolapse=fothergill's operation (manchester)

72- Girl Every month there is exacerbation sob relieved by nebulized in emergency
what to add?
A. add inhaler budesonide
B. chromylyn
C. slow theophylline
D. oral steroids

• Correct Answer:A

30
73- A girl who is on low mood, increased appetite, sad, irritated that usually
happens before her menstruation & she has abdominal pain associated with her
menstruation that affects her social life, what treatment to put her on?
A. OCP
B. selective serotonin uptake inhibitor

• Correct Answer:B

UpToDate: Targeting the serotonin system by increasing central serotonergic


transmission (selective serotonin reuptake inhibitors [SSRIs]). Of all the treatment
options for PMDD, SSRIs have the best evidence for efficacy.

Extra Notes: (Drugs = Antidote)


-Acetaminophen (paracetamol)➡️ 'N-Acetyl Cystine (NAC)
-Beta Blockers➡️ Glucagon
-Opioids➡️ Naloxone
-Anticholinergic➡️ Physostigmine
-Benzodiazipines--➡️Flumazenil

31
-Warfarin--' ➡️FFP (early) or Vit K (late)
-Heparin ➡️Protamine sulfate
-Insulin--➡️ Glucagon
-Iron---➡️ Desferoxamine
-Digoxin ➡️Digoxine immune Fab
-CCB ➡️Calcium chloride
-Mg+2 Sulfate ➡️Calcium gluconate
-ASPRIN➡️sodium bicarbonate

74- female pregnant with hypertension and proteinuria, she has right upper
quadrant pain what is the reason?
A- Distended Hepatic Capsule
B- Hepatic Rupture
C- Gallbladder Stone

• Correct Answer:A

UpToDate: In preeclampsia the liver may be tender to palpation due to stretching


of Glisson's capsule from hepatic swelling. It is characterized by severe constant
pain that often begins at night, usually maximal in the low retrosternum or
epigastrium, but may radiate to the right hypochondrium or back.

75-57 y/o woman has 3rd degree hemorrhoids (with NO bleeding) which of the
following is the appropriate management?
A. fiber supplementation
B. rubber band ligation
C. surgical hemorrhoidectomy

• Correct Answer:B (UpToDate:), C (Dr. Abeidi)

UpToDate: For patients with symptomatic grade I, II, or III internal hemorrhoids
refractory to conservative treatment, we recommend an office-based procedure.
For grade III hemorrhoids, one can first try rubber band ligation, which is often
successful after one or two applications rather than surgical hemorrhoidectomy, as
the initial intervention.

32
76- Blood loss of 25%, what to expect to be decreased?
A- RR
B- Pulse pressure
C- GCS scale
D- Urine output

• Correct Answer:B

UpToDate: The Advanced Trauma Life Support (ATLS) manual describes four
classes of hemorrhage to emphasize the early signs of the shock state.
●Class I hemorrhage involves a blood volume loss of up to 15 percent. The heart
rate is minimally elevated or normal, and there is no change in blood pressure,
pulse pressure, or respiratory rate.
●Class II hemorrhage occurs when there is a 15 to 30 percent blood volume loss
and is manifested clinically as tachycardia (heart rate of 100 to 120), tachypnea
(respiratory rate of 20 to 24), and a decreased pulse pressure, although systolic
blood pressure changes minimally if at all. The skin may be cool and clammy, and
capillary refill may be delayed.
●Class III hemorrhage involves a 30 to 40 percent blood volume loss, resulting in a
significant drop in blood pressure and changes in mental status. Heart rate (≥120

33
and thready) and respiratory rate are markedly elevated, while urine output is
diminished. Capillary refill is delayed.
●Class IV hemorrhage involves more than 40 percent blood volume loss leading to
significant depression in blood pressure and mental status. Most patients in class
IV shock are hypotensive (systolic blood pressure less than 90 mmHg). Pulse
pressure is narrowed (≤25 mmHg), and tachycardia is marked (>120). Urine output
is minimal or absent. The skin is cold and pale, and capillary refill is delayed.

Important DDx of Vaginal Conditions 🛑


-Dyspareunia +/- dysuria + frequency in > 51 YO (+/- vaginal itching /dryness) ➡️
suspect Atrophic vaginitis (Topical estrogen cream).

- Itchy, tender white plaque of the vulva (become itchier at night) ➡️ Lichen
Sclerosis (Topical steroids | Follow up).

- White Thick discharge ➡️ Candida (Vaginal Thrush) (Topical clotrimazole).

- Yellow-greenish offensive discharge + Vaginal itching +/- Strawberry Cervix +/- pH


> 4.5 ➡️ Trichomonas Vaginalis (Trichomoniasis) (Metronidazole).

- Offensive discharge WITHOUT itching +/- fishy smell +/- pH > 4.5, Whiff’s test
positive ➡️ Bacterial Vaginosis (Gardnerella Vaginalis) (Metronidazole).

Pictures:

Falciparum malaria Rooting reflux

34
ABSENT RED REFLEX ANEMBREONIC SAC

ENDOMETRIOSIS BASAL CELL CANCER

35
77- Man wants to travel and seeking travelers’ diarrhea prophylactic treatment, his
labs show abnormal KFT. What treatment to give?
A- Ciprofloxacin
B- Bismuth
C- No need
D- Probiotics

• Correct Answer:C

UpToDate: Antibiotic chemoprophylaxis should not be used routinely for travelers.


Nonantibiotic agents (Bismuth subsalicylate) can prevent a significant number of
cases of travelers' diarrhea. However, the doses required are inconvenient for the
traveler, and salicylate toxicity is a potential complication.

78- Child c/o fever, bloody stool and tenesmus, abdominal examination showed
abdominal distention, Dx?
A. Ascaris
B. Amebiasis
C. Giardiasis
D. Rotavirus

• Correct Answer:B

AMBOSS: Amebiasis: Loose stools with mucus and bright red blood, Painful
defecation, tenesmus, abdominal pain, cramps, weight loss, and anorexia.
Ascaris: early symptoms: dry cough, blood-tinged sputum, wheezing.
Late symptoms: anorexia, abdominal discomfort, nausea, vomiting, and diarrhea.
Rota: Abdominal pain, vomiting and watery diarrhea.
Giardiasis: diarrhea foul-smelling, voluminous, frothy, and fatty stools (stools tend
to float and do not appear bloody).

79- Pediatric case h/F sore throat & fever then 3 days he develops body rash on
the thighs and buttocks. Lab finding suggest renal insult with anemia and normal
Plat count) plus ll edema +black tea urine.

36
A -post streptococcal GN
B -HUS
C -HSP (Henoch-Schonlein Purpura)

• Correct Answer:C

AMBOSS: Symmetrically distributed,


raised, erythematous macules or urticarial lesions that coalesce
into palpable purpura (nonblanching skin lesions). Most common sites: the lower
extremities, buttocks.

80- According to celiac disease what is the appropriate to confirm diagnosis?


A- Biopsy
B- ani tans and ani endo
C- Gluten free diet and relieve of symptoms

• Correct Answer:A

AMBOSS: Diagnosis is based on serology (initial testing)


and EGD with duodenal biopsy (confirmation).

37
81- 40 years old diabetes male comes with painful perianal swelling with fever for
the last 3 days. T 38 , WBC was 16 , neutrophil was 80 , random glucose was 6.5.
What’s the DX ?
A.Anal fissure
B.Perianal abscess
C.Hemorrhoid
D.Thrombosed piles

• Correct Answer:B

➔ Ruptured abscess >> fistula


➔ Abscess >> mass or swelling, pain, tenderness, redness and purulent discharge,
usually with fever or leukocytosis.
➔ Hematoma >> painful mass only.
➔ Hemorrhoids >> painless, prolapsed mass with bleeding; becomes painful when
thrombosed.
➔ Fissure >> pain (during and after defecation) bleeding.

82- Elderly female with Hx of COPD came to you complaining of SOB DYSNPEA She
is using LABA, LAMA, SABA, ICS and i think other medications , she did chest
physiotherapy Vitals are stable ABG normal O2%90-93% PH normal Co2 normal As
much as i remember everything was normal What is the next step?
A- repeat physiotherapy
B- add oral steroids
C-long term O2 therapy
D- Mechanical ventilation

• Correct Answer:B

UpToDate::For outpatients with a COPD exacerbation characterized by


breathlessness that interferes with daily activities, systemic glucocorticoid therapy
appears to have a small but beneficial effect with a reduction in rate of relapse.

83- surgeon could not control bleeding and discuss it with the assistant consultant
and they decide to do hysterectomy to save patient life?

38
A-procede with hysterectomy without consent
B-Be limited on what was mentioned in the consent
C-Take husband consent
D-wait for consent or something?

• Correct Answer: A

84-Drug given for long therapy of esophageal varices:

• Correct Answer: BB

AMBOSS: primary prophylaxis: Medication to lower portal pressure, irrespective of


variceal grading: nonselective beta-blockers (e.g., propranolol, nadolol).

85-Abruption placenta 32wks and bleeding stopped your action??


A-admission and follow up in hospital
B-discharge and normal follow-up
C-high risk and follow-up with serial u/s

• Correct Answer: A

UpToDate: Hospitalization – There are no compelling data to guide the length of a


hospital stay for these patients. A reasonable approach is to monitor the patient in
the hospital until the bleeding has subsided for at least 48 hours, fetal heart rate
tracings and ultrasound examinations are reassuring, and the patient is
asymptomatic. At that point, discharge may be considered. Importantly, the
patient should be counseled to return immediately if they have more bleeding,
contractions, decreased fetal movement, or abdominal pain. In patients with
sonographic evidence of a large hematoma, we believe it is prudent to keep the
patient in the hospital for a longer period for close monitoring.
Kaplan Obstetrics: Management of Abruptio Placentae:
1-Emergency cesarean delivery is performed if maternal or fetal jeopardy is
present as soon as the mother is stabilized.
2-Vaginal delivery is performed if bleeding is heavy but controlled or pregnancy is
>36 weeks. Perform amniotomy and induce labor. Place external monitors to
assess fetal heart rate pattern and contractions. Avoid cesarean delivery if the
fetus is dead.

39
3-Conservative in-hospital observation is performed if mother and fetus are stable
and remote from term, bleeding is minimal or decreasing, and contractions are
subsiding. Confirm normal placental implantation with sonogram and replace
blood loss with crystalloid and blood products as needed.

Uterine fibroid

86- 37 Y/O Female pt with family Hx of mother had breast cancer & sister had
ovarian cancer which screening test is appropriate for her?
A) Annual breast mammogram starting from now
B) Annual breast mammogram starting at age of 40

• Correct Answer:A

UpToDate: The following screening strategy is recommended for women


with BRCA1/2 pathogenic variants who have not undergone risk-reducing surgery,
and should be individualized as needed:

• Magnetic resonance imaging (MRI) for breast cancer screening is


recommended annually beginning at age 25, or earlier depending on the
earliest age of breast cancer in the family.
• Mammography with consideration of tomosynthesis should begin at age 30
or be individualized if the earliest age of onset in the family is under age 25.
Breast MRIs and mammography may be staggered by 6 months.
NOTE: But the best is BRCA MUTATION TEST.

87- Health organization “good health for all” the year 2030 will be the end of
preventable death for children under 5 y/o by?

40
A. School health
B. Health education
C. Immunization ion

• Correct Answer:C

88-14 y/o boy ingested a bottle of acetaminophen tablets brought by his parents
20 hour later, asking about which stage of toxicity?
A-1
B-2
C-3
D-4

• Correct Answer:A

UpToDate: ●Stage I (up to 24 hours after overdose) – Asymptomatic but less


commonly: nausea, vomiting, and, in patients with very large doses, lethargy and
malaise
●Stage II (24 to 72 hours after overdose) – Right upper quadrant pain, elevation in
liver enzymes, prothrombin time (PT) and international normalized ratio of PT, and,
in severe cases, evidence of nephrotoxicity (elevated blood urea nitrogen,
creatinine, oliguria) and/or pancreatitis (elevated serum amylase, lipase)
●Stage III (72 to 96 hours) – Evidence of liver failure and, in severe cases, renal
failure and multi-organ failure; death most commonly occurs in this stage
●Stage IV (4 to 14 days) – Recovery

89- An 8 years old girl ingested 2 pack of paracetamol after a fight with her mother
come after a 24 of ingestion with RUQ pain in which stage of toxicity she is?
A- Stage 1
B-Stage 2
C-Stage 3
D-Stage 4

41
• Correct Answer:B

90-Which of the following medications, when used alone as maintenance Therapy


in persistent asthma, is associated with an increased risk of asthma-related
mortality?
A- Inhaled fluticasone
B- Inhaled salmeterol
C- Oral zafirlukast
D- Oral prednisone

• Correct Answer:B

American Academy of Family Physicians: Salmeterol also is associated with


increased risk of death from asthma in patients who are not taking inhaled
corticosteroids, and, therefore, cannot be used as a replacement for inhaled
corticosteroids for long-term asthma control.

91- Timing of cleavage for monochorionic monoamniotic twins?

• Correct Answer:8-12 days

AMBOSS:

42
92-Otitis media most common organism viral?
A-rhinovirus
B-rotavirus
C-coronavirus

• Correct Answer:A

UpToDate: rhinovirus and respiratory syncytial virus were most commonly


identified.
First RSV, then rhinovirus.

93- 12 year-old received a nonspecific blunt trauma on his abdomen and later
presented with generalized abdominal pain. Imaging of the spleen showed a 7 mm
hematoma and 4 cm tear (grade 3) Your management?
A. splenectomy
B. Spleen preserving surgery
C. Conservative

• Correct Answer:C

UpToDate: The management is dictated by the clinical course, not by the injury
grade or the grade of injury present on the CT. Patients who remain
hemodynamically stable can be treated nonoperatively. This is possible in >90
percent of cases.

94-A patient is admitted to ICU and intubated because of a shock. Which of the
following is indicating systemic perfusion?
A-cardiac index
B-central venous pressure
C-Mixed venous oxygen saturation
D-Central venous oxygen saturation

43
• Correct Answer:D

UpToDate: A true mixed venous sample (called SvO2) is drawn from the tip of the
pulmonary artery catheter. Because pulmonary artery catheter use has declined
dramatically, ScvO2 measurements obtained from internal jugular or subclavian

catheters are often used and interpreted in the same manner. An ScvO2 refers to a
central venous sample. An ScvO2 measurement is a surrogate for the SvO2.

95- A pregnant in 8 weeks GA, US showing a fetus without viability. What is the
appropriate management?
A- misoprostol
B- mifepristol
C- surgical
D- conservative management

• Correct Answer: D

UpToDate: Expectant management is another phrase for watchful waiting‫ آ‬for the
pregnancy tissue to pass on its own. In the first trimester, efficacy rates of 70 to 80
percent have been reported for this approach. Beyond 13 weeks gestation, we
advise either medication management in a health facility or surgical management.

96- Drug to be safely given to CKD 3rd stage:


A-Metformin
B-Nitrophorin
C-Warfarin
D-Lithium

• Correct Answer:C

UpToDate: -Metformin CI: impaired kidney function (estimated glomerular


filtration rate [eGFR] <30 mL/min/1.73 m2).
- Nitrofurantoin CI: anuria, oliguria, or significant impairment of renal function.
- Lithium is associated with chronic renal toxicity.

97- What is the absolute side effects of amitriptyline?

44
A- weight gain
B- Constipation
C- Insomnia

• Correct Answer:C

PubMed: These findings show that chronic administration of low-modest doses of


tricyclic antidepressants frequently cause considerable weight gain and can
significantly interfere with the ability to provide long-term maintenance therapy.

98- Which one of the following drugs is a selective beta-1 agonist?


A- Amrinone
B- Digoxin
C- Dobutamine
D- Dopamine

• Correct Answer:C

UpToDate: C) beta-1 agonist used to treat cardiac decompensation in patients with


organic heart disease or from cardiac surgery.
A) Amrinone is a positive inotropic agent and phosphodiesterase inhibitor used in
the management of treatment of congestive heart failure.
B) It is a common agent used to manage atrial fibrillation and the symptoms of
heart failure.
D) A catecholamine neurotransmitter used to treat hemodynamic imbalances,
poor perfusion of vital organs, low cardiac output, and hypotension.

99- Pregnant with sever preeclampsia which of the following is significantly


decrease (written like this not increase I read it several times to make sure ):
A) Serum urea
B) plasma volume
C) Serum creatinine

45
D) Serum uric acid
NO platelets

• Correct Answer:B

PubMed: Plasma volume is reduced in women with pre-eclampsia and this


reduction is associated with a low concentration of serum albumin.

100- Scenario of pt alcholic presented with jaundice fatigability only no fever


otherwise asymptomatic and on examination you found soft smooth enlarged liver
span 13 cm
No splenomegaly
There was a list of labs investigation:
AlT.,AsT mildly elevated
Alp mildly elevated
Bilirubin high
Iron was high
Ferretin was very high 450 compared to Normal in exam
HCV negative
HBsAg negative
HBsAb positive
What is the most likely diagnosis?
A) chronic Hepatitis B
B) alcoholic Hepatitis
C) hemochromatosis

Correct Answer:C
AMBOSS: ↑ Serum iron
• ↑ Ferritin in serum (> 200 μg/L)

46
• ↓ Total iron-binding capacity (TIBC)
• ↑ Transferrin saturation (> 45%)
• ↑ Liver enzymes

101-Child had pneumonia (labs show leukocytosis) and on oral Abx for one week
mother till his symptom improved., what next:
A- No need more investigation.
B- Culture and sensitivity

• Correct Answer:A

UpToDate: Children with CAP who are appropriately treated generally show signs
of improvement within 48 to 72 hours. Follow-up investigation/radiographs are not
necessary in asymptomatic children with uncomplicated CAP.
102- A diabetic patient presented with exudates from a wound in his leg with
poor healing and no sign of inflammation. What is the cause of the poor wound
healing in this diabetes melilites patient?
A. Decreased phagocytosis process.
B. Stimulated bacterial growth.
C. Decreased immunity.
D. Increased blood supply to the wound

• Correct Answer:A

UpToDate: Numerous cytologic factors contribute to impaired wound healing in


patients with diabetes.These include decreased or impaired growth factor
production, angiogenic response, macrophage function…

103- Patient with multiple joint ache, Malar rash, positive Anti Smith. C3 and C4
positive, creatinine high, liver functions high too. Treatment?
A. Chloroquine n methotrexate
B. Chloroquine and MMF

47
C. Methotrexate and ibuprofen

• Correct Answer:B

UpToDate: Patients with severe or life-threatening manifestations secondary to


major organ involvement (eg, renal and central nervous system) generally require
an initial period of intensive immunosuppressive therapy (induction therapy) to
control the disease and halt tissue injury. Patients are usually treated for a short
period of time with high doses of systemic glucocorticoids (eg, intravenous
"pulses" of methylprednisolone, 0.5 to 1 g/day for three days in acutely ill patients,
or 1 to 2 mg/kg/day in more stable patients) used alone or in combination with
other immunosuppressive agents. There is a paucity of data to support the use of
intravenous "pulse" versus daily oral glucocorticoids [100]. One of the major
advantages of glucocorticoid therapy is that it rapidly reduces inflammation,
thereby helping to achieve disease control. However, attempts to minimize
exposure must always be made, given the long-term adverse effects. (See "Major
side effects of systemic glucocorticoids".)

Examples of other immunosuppressive agents that may be used include


mycophenolate, azathioprine, cyclophosphamide, or rituximab. This initial therapy
is subsequently followed by a longer period of less intensive, and ideally less toxic,
maintenance therapy to consolidate remission and prevent flares. During this
phase of treatment, the dose of prednisone or equivalent is reduced while
monitoring clinical and laboratory measures of disease activity.

104- Long case of COPD patient with oxygen saturation 88% and PHTN is 50 What
improve survival?
A) Long term oxygen
B) oral steroid
C) overnight pulse oximeter

• Correct Answer:A

48
PubMed: The following can improve COPD mortality: smoking cessation; long-
term oxygen therapy; and lung volume reduction surgery in small subsets of COPD
patients.

105- Trauma patient presented with a wound on his thigh, subcutaneous fat is lost
but vasculature is intact. What provides the best management?
A. Debridement with primary closure
B. Primary repair
C. Debridement with secondary closure
D. Debridement with vacuum-assisted closure

• Correct Answer:C

UpToDate: Secondary intention — Indications for secondary closure (ie, by


granulation) include:●Deep stab or puncture wounds that cannot be adequately
irrigated
●Contaminated wounds
●Abscess cavities
●Presentation after a significant delay (eg, >24 hours)
●Noncosmetic animal bites

-Vascular intact
SO closure with graft
‫ لو قال‬necrotic skin ‫ نختار‬debridment with vacuum
‫لو قال‬scalp laceration ‫ نختار‬debridment with primary closure

106: 30y male came with dyspnea and chest pain PND orthopea O/E pansytolic
mumur gallop s3 with displaced apex no any significant medical hstiory most
appropritate next step?
A.chest xray
B.ECG
C.Echo

• Correct Answer:B

49
UpToDate: The diagnosis in patients with suspected MR is generally confirmed by
transthoracic echocardiogram (TTE). TTE is recommended in patients with known
or suspected MR to determine the severity, etiology, and hemodynamic
consequences of MR. but the next step is ECG. But the next step is ECG.

107- Parents brought their child to the hospital for inability to move his arm xray
showed fracture at both distal ulna and radial bone. The child looks healthy and
dressed well, and good
hygine they recall that 2 days ago the mom droped a plate at the child arm and
noticed he
couldn’t move his arm and after 2nd day the child couldn’t move his arm they
didn’t notice
any brusing so they thought it’s just a strain. Which of the following is indicative of
child
abuse.
A. Type of fracture
B. Delayed presentation
C. Consistent story of the parents
D. the appearance of the baby

• Correct Answer:B

UpToDate: Delay in seeking care — Delays that complicate care, prolong pain, or
that occur in children with obvious, severe distress (eg, actively seizing, coma, or
respiratory distress) are suspicious for abuse.

108-A child with pica, hepatosplenomegally and failure to thrive. Coming from a
low socioeconomic economic status family.
HGB low
Lead - 2 high
PT normal
PTT normal
INR normal
Iron normal
X-ray revealed bands of increased metaphyseal

50
What’s the most appropriate treatment?
A. Vitamin K
B. D-pinicillamine
C. Iron supplements

• Correct Answer:B

UpToDate: American Academy of Pediatrics (AAP) guidelines for the treatment of


lead toxicity describe penicillamine as a third-line agent, indicated only when
unacceptable reactions have occurred to succimer or CaNa2EDTA, and continued
therapy is required. Iron supplementation should not occur during chelation
therapy.
The role of iron supplementation in children with lead poisoning who are not
receiving chelation is unclear. Like calcium, dietary iron is thought to decrease the
intestinal absorption of lead.

109- Supracondylar fracture case with no pulsation (not mentioned cold or warm).
A- k wire
B- surgical exploration

• Correct Answer:A

UpToDate: The emergency clinician should promptly identify children with vascular
insufficiency and obtain an emergency consult with an orthopedic surgeon with
appropriate pediatric expertise. Rarely, these children will require partial closed
reduction in the emergency department to restore distal circulation prior to closed

or open reduction and stabilization in the operating room. Patients who display a
cold, white or cyanotic hand despite reduction attempts warrant operative
exploration of the brachial artery and vascular repair.

51
110- 9 month came to clinic with fever irritability disturbance of sleep on exam
performed drum with pus in external canal?
A. rotavirus
B. rhinovirus
C. adenovirus
D. coronavirus

• Correct Answer:B

UpToDate: rhinovirus and respiratory syncytial virus were most identified in otitis
media.

111- A patient underwent sigmoidoscopy found sigmoid lesion, histology report:


Sigmoid Adenocarcinoma with free margin with no invasion to mucosa. What is the
next step?
A- Abdominal CT scan
B- Sigmoidectomy
C- Pelvic MRI
D- PET scan

52
• Correct Answer:A

UpToDate: In the United States and elsewhere, the standard practice at most
institutions is that all patients with stage II, III, or IV CRC undergo chest, abdomen,
and pelvic CT, either prior to or following resection, an approach endorsed by the
National Comprehensive Cancer Network.
Note: A. for staging
B. if the pt came with obstruction

112- Screening for 37 female her mother had brest Ca, sister ovarian Ca:
A- breast US
B- mammogram
C- BRCA

• Correct Answer:C

UpToDate: Most hereditary breast and ovarian cancers are due to highly penetrant
germline BRCA1/2 pathogenic variants, which are inherited in an autosomal-
dominant fashion.

113- Patient did cardiac bypass, then he c/o abd. High cardiac output, low
peripheral vascular resistance?
A- cardiac shock
B- septic shock

53
• Correct Answer:B

114- BIRAD IV ?

• Correct Answer:core biopsy.

UpToDate: All reports with BI-RADS 0, 4, or 5 need further intervention. In most


institutions, the clinician is contacted to convey the need for biopsy

115-Phylloid What is the management?

UpToDate: A complete surgical excision is the standard of care for phyllodes


tumors. Current National Comprehensive Cancer Network (NCCN) guidelines
recommend excisional biopsy for benign phyllodes tumors and wide excision with
intention of obtaining surgical margins of 1 cm or more for borderline and

malignant phyllodes tumors.


Mastectomy is generally not indicated for benign phyllodes unless the tumor is so
large that breast-conserving surgery would result in suboptimal cosmetic
outcomes.

116- Neuropathy pain TX?

• Correct Answer:amitriptyline

UpToDate: Several TCA drugs (amitriptyline, desipramine, nortriptyline) are


effective for neuropathic pain in patients with diabetic neuropathy and can be

54
used cautiously for this purpose, with attention to their higher risk of serious side
effects.

117- Patient with sjogren syndrome which type of RTA: + Labs: Shows low K
Normal Na ?
A) RTA I
B) RTA II
C) RTA III
D) RTA IV

• Correct Answer:A

UpToDate: A number of different defects can directly or indirectly decrease the


net activity of the distal tubule proton pump (eg Sjögren's syndrome )

118- Symmetrical uterus enlargement?

• Correct Answer:adenomyosis

55
UpToDate: The bimanual pelvic examination in patients with adenomyosis typically
shows a uterus that is mobile, diffusely enlarged (often referred to as "globular"
enlargement), and soft (often referred to as "boggy").

119- Aspergillosis Tx?

• Correct Answer:voriconazol

UpToDate: Voriconazole should be included in the antifungal regimen in most


patients with invasive aspergillosis.

120- Patient go for umrah and s&s of meningitis?


A- ceftrixone + Azethromycine +steroid
B- Azethromycine + ceftrixone
C-Vancomycin
D-ceftrixone +vancomycin +Dexa

• Correct Answer:D

UpToDate: Antimicrobial therapy, along with adjunctive dexamethasone, should be


initiated as quickly as possible. Selected third-generation cephalosporins
(ie, ceftriaxone) are the beta-lactams of choice in the empiric treatment of
meningitis. In countries with ceftriaxone resistance rates >1
percent, vancomycin should be added.

121- Pregnant with HTN on hydrothiazide + ARBs ?


A- stop all medication
B- continue all medication

56
C- stop ARBs give methyldopa

• Correct Answer:A

AMBOSS: Contraindicated in pregnanacy: ACEIs, ARBs, renin inhibitors


(e.g., aliskiren), or atenolol. Diuretics should only be initiated in pregnancy for
treatment of hypertension when clinically indicated for hypertension associated
with volume overload (chronic kidney disease or heart failure), or when other
agents have not been successful in achieving blood pressure targets.

122- Prosthetic valve prophylaxis?

• Correct Answer:Vancomycin + gentamycine + rifampicine

UpToDate: empiric broad-spectrum antibiotic therapy to cover both gram-positive


and gram-negative bacteria should be initiated: vancomycin, gentamicin (for
synergy) .Rifampin appears to be unique in its ability able to kill staphylococci
adherent to foreign material.

123- 29 y/o male with few days history of Seizure and confusion, and Hx of
unprotected sexual intercourse for 6 years. WBC 1.4. What is the next step?
A- HIV serology
B- CSF analysis for syphilis

57
C- CSF analysis for toxoplasma
D- Herpes pcr

• Correct Answer:A

Note: Choose (A) if positive cryptococcal + low WBC


(B) if they didn’t mention the above

124- Old man came complaining that (he can’t remember things), he sometimes
forgets his friends’ names or celebrities in his community and phone numbers. His
wife is worried he has Alzheimer’s. Labs normal. He has tender knees (not
something amazing just cause he’s old). No mention how it affects his daily life.
A- Alzheimer’s
B- Benign forgetfulness

• Correct Answer:B

UpToDate: criteria for probable Alzheimer’s dementia require the presence of


dementia and the following characteristics:
•Interference with ability to function at work or at usual activities
•A decline from a previous level of functioning and performing
•Not explained by delirium or major psychiatric disorder
•Cognitive impairment established by history-taking from the patient and a
knowledgeable informant; and objective bedside mental status examination or
neuropsychologic testing
•Cognitive impairment involving a minimum of two of the following domains:
-Impaired ability to acquire and remember new information
-Impaired reasoning and handing of complex tasks, poor judgment
-Impaired visuospatial abilities
-Impaired language functions
-Changes in personality, behavior, or comportment
•Other core clinical criteria include:
-Insidious onset
-Clear-cut history of worsening
-Initial and most prominent cognitive deficits are one of the following:
amnestic presentation (ie, impairment in learning and recall of recently
learned information) or nonamnestic presentations (including either a
language presentation, with prominent word-finding deficits; a

58
visuospatial presentation, with visual cognitive deficits; or a
dysexecutive presentation, with prominent impairment of reasoning,
judgment, and/or problem solving).

125- A baby diagnosed with Cystic fibrosis. He has positive sweat chloride test his
brother is normal. To confirm diagnosis of cystic fibrosis?
A. CFTR gene in parent
B. CFTR in sibling
C. Chloride test in parent
D. Chloride test in sibling

• Correct Answer:B

UpToDate: Clinical symptoms consistent with CF in at least one organ


system, or positive newborn screen or having a sibling with CF AND

59
●Evidence of cystic fibrosis transmembrane conductance regulator (CFTR)
dysfunction (any of the following):
•Elevated sweat chloride ≥60 mmol/L
•Presence of two disease-causing mutations in the CFTR gene, one from
each parental allele
•Abnormal NPD

126- Patient diagnosed with thalassemia major and you decide to start regular
blood transfusion. Which of the following is the most important vaccine?
A. Hepatitis A
B. Hepatitis B
C. DTP
D. MMR,
E. Influenza

• Correct Answer:B

AMBOSS: Despite the low risk of transmission of hepatitis B after routine


screening, hepatitis B vaccination is recommended for patients who require
frequent transfusions.

127- Child came with pallor and jaundice. His father and grandfather are both
known to have chronic hemolysis and underwent gallbladder removal. What is the
best test for diagnosis?
A. Hb electrophoresis,
B. Sickle cell test
C. Coombs test
D. Osmotic fragility test

• Correct Answer:D

UpToDate: The majority of affected individuals of hereditary spherocytosis have


mild or moderate hemolysis or hemolytic anemia and a known family history,
making diagnosis and treatment relatively straightforward. EMA binding is our
preferred test, and it is widely available. We generally order EMA binding. If the
results are equivocal, the osmotic fragility test or osmotic gradient ektacytometry
(if available) can be added.

60
128- 40s old female, with heavy bleeding came to ER, what to do?
A- IUD
B- D/C
C- Hysterectomy
D- Mefenamic acid

• Correct Answer:B

UpToDate::For hemodynamically unstable patients with acute uterine bleeding,


uterine curettage is the treatment of choice.

129- A patient with liver cirrhosis presents with worsening deterioration in


consciousness for some of days, examination revealed Tense ascites.
Albumin 30
Bilirubin 30
INR 1.9
What class of the Child Pugh?
A. Class A
B. Class B
C. Class C
D. No classification

• Correct Answer:C

61
130- Young female (not pregnant) with hx of PE 1 year ago, presented with swollen
leg, Us showed proximal DVT, most appropriate management?
A- long term anticoagulant therapy
B-anticoagulant for 2 weeks
C-thrombolysis
D- IVC

• Correct Answer:A

UpToDate: Anticoagulant therapy is indicated for all patients with proximal DVT,
regardless of the presence of symptoms and provided there is no contraindication
to anticoagulation.

131- Pt did let neck surgery then develop numbness (or loss of sensation) in the
lower part of pinna + left triangle of mandible. Which nerve is injured?
A. Third occipital

62
B. Great auricular
C. Great occipital
D. Lesser occipital

• Correct Answer:B

132- what could be cured after splenectomy?


A-ITP
B-TTP
C-Thalassemia

• Correct Answer:A

UpToDate: Splenectomy may be a good choice for an individual who wishes to


have a single potentially curative surgical procedure and who is willing to accept
the small increased risks of infection and venous thromboembolism.
Splenectomy, rituximab, or a thrombopoietin receptor agonist (TPO-RA) are the
three principal choices of second-line treatment; they differ in their mechanism of
action.

133- Infant Mortality Rate has decreased to 4.28 in 2020 in Saudi Arabia.
Which of the following figures involved in This:
A) Number of women in childbirths age
B) Number of deaths in saudi arabia in 2020
C) children who died in 2020 and not completed their first year

• Correct Answer:C

134- 8 YO female presents to the clinic after finding of hilar lymphadenopathy on


CXR. She has on and off cough, but denies any fever, headache, n/v, weight loss all
negative. Labs insignificant except for hypercalcemia X-ray shows confirmed

63
bilateral hilar lymphadenopathy CT guided biopsy shows noncaseating granuloma
What is the best next step in management?
A-Observe
B-Start prednisolone
C-Start azithromycin
D-Start anti TB medication

• Correct Answer:B

UpToDate: When assessing the indications for glucocorticoid therapy, it is helpful


to keep in mind that certain features help to predict the likely course of disease. A
longer period of watchful waiting prior to initiating therapy may be possible in
patients with more favorable characteristics. Favorable characteristics include
stage I disease, especially when associated with erythema nodosum, absence of
symptoms, and European descent. Unfavorable prognostic features, in contrast,
include the presence of symptoms (especially dyspnea), multisystem involvement
(three or more organs involved), increasing chest radiographic opacities,
extrapulmonary disease. Oral glucocorticoid therapy generally improves symptoms
(eg, cough and dyspnea) and radiographic abnormalities.

135- 2.5 y/o boy keeps yelling NO (and some other like this) what will you advise
the parents?
A- Ignorance
B- Counseling
C- positive reinforcement

• Correct Answer:C

UpToDate: Social learning begins at an early age through imitation and positive or
negative reinforcement of behaviors.

136- Appendix perforation antibiotic coverage?


A.Positive aerobes
B.Positive anerobes

64
C.Negative aerobes
D.Negative anerobes and aerobes

• Correct Answer:D

UpToDate: In patients with perforated appendicitis, the antibiotic regimen should


consist of empiric broad-spectrum therapy with activity against gram-negative rods
and anaerobic organisms pending culture results.

137- Hepatitis A vaccine after blood transfusion?


A-As schedule
B-After 3 m

• Correct Answer:A

Note: As scheduled because recent blood transfusion is not a contraindication to


killed vaccines such as hep A

138- A new screening test for diabetes has a sensitivity of 90% and specificity of
80%
Which of the following is the best interpretation?
A. 80% of non-diabetics had positive result
B. 80% of diabetics had positive result
C. The test was positive for 90% of diabetic patients
D. For each 100 positive test results, 90% were diabetics

• Correct Answer:C

UpToDate: Sensitivity is the number of patients with a positive test who have a
disease divided by all patients who have the disease. A test with high sensitivity will
not miss many patients who have the disease (ie, few false-negative results).
Specificity is the number of patients who have a negative test and do not have the
disease divided by the number of patients who do not have the disease. A test with
high specificity will infrequently identify patients as having a disease when they do
not (ie, few false-positive results).

65
139- Perinatal mortality:
a) Includes all stillbirth after the 20th week of pregnancy
b) Includes all neonatal deaths in the 1st 8 week of life
c) Includes all stillbirths & 1stweek neonatal deaths
d) Specifically neonatal Deaths.
e) Is usually death per 10,000 live births

• Correct Answer:C

UpToDate: Three different definitions are used for perinatal death, which vary
globally:
●Definition 1 – Sum of infant deaths that occur at less than 7 days of age and
fetal deaths (stillbirths) with a gestational age of 28 weeks or more.
●Definition 2 – Sum of infant deaths that occur at less than 28 days of age
and fetal deaths with a gestational age of 20 weeks or more.
●Definition 3 – Sum of infant deaths that occur at less than 7 days of age and
fetal deaths with a gestational age of 20 weeks or more.

140- Which of the following is not associated with cystic fibrosis?


A. Sinusitis
B. Lung cancer
C. Pancreatic insufficiency
D. Small bowel obstruction

• Correct Answer:B

AMBOSS: By exclusion. The clinical presentation of CF includes all the choices


except lung cancer.

66
141- Which of the following tests is used to establish a diagnosis of acromegaly ?
A. Liver function tests (LFTs)
B. Complete blood count (CBC)
C. Oral glucose tolerance test (OGTT)
D. Hemoglobin A1C (HbA1c)
E. Thyroid function tests (TFTs)

• Correct Answer:C

AMBOSS: OGTT with baseline GH and measure GH after 2 hours: the most specific
test.

142. 2 y/o pt on chemo for ALL, contacted a chicken pox pt, what to do?

A. Acyclovir 7days
B. Give vaccine now
C. Stop chemo
D. Give varicella zoster Immunoglobulin

• Correct Answer:D
UpToDate: Passive immunization with varicella-zoster immune globulin (eg,
Varizig) is indicated for susceptible individuals after a significant exposure if they
are ineligible for varicella vaccine prophylaxis, at high risk for severe infection
and/or complications, and can receive immunoprophylaxis within 10 days of
exposure.
Indications:
• Immunocompromised patients of any age who lack evidence of immunity to
VZV.
• Pregnant women who lack evidence of immunity to VZV.
• Newborns of mothers who develop varicella five days before to two days
after delivery.
• Hospitalized premature infants born at ≥28 weeks of gestation whose
mothers do not have evidence of immunity.

67
• Hospitalized premature infants born at <28 weeks of gestation or who weigh
≤1000 g at birth, regardless of maternal evidence of immunity to varicella.

143. Man with 3 months hx of epigastric pain, With loss of weight and appetite ,
most investigations?
A. Endoscopy
B. CT

• Correct Answer:A
AMBOSS: Perform a clinical evaluation, focusing on red flags in GERD and
evaluating dyspepsia if present
Rule out life-threatening differential diagnoses of GERD and chest pain (e.g., acute
coronary syndrome)
Typical symptoms without red flags in GERD: Initiate treatment for GERD; start an
empiric once-daily PPI trial.
If there is relief after 8 weeks of PPI: GERD is likely; PPI can be discontinued.
If symptoms persist on PPI or recur after discontinuing PPI: Refer to
gastroenterology for EGD
Red flags in GERD: Refer to gastroenterology for EGD

144. Pregnant with warts in vulva treatment?


A. Cryotherapy
B. electrophoresis

• Correct Answer:A
AMBOSS:
Treatment
In most cases the infection clears up without any treatment. Treatment options of
HPV–related anogenital warts are: Routine clinical monitoring or Local treatment
with one of the following:
Podophyllotoxin, Imiquimod, Trichloroacetic acid, Cryotherapy, or Surgical
removal.
Evidence of malignancy should always be excluded on HPV–related cervical lesions
via cytological and histological monitoring.

68
145. Patient with epilepsy, most common shoulder dislocation:
A- inferior
B- supraacromine posterior

• Correct Answer:Anterior (incomplete question if there was no anterior pick


posterior)
Bone and joint journal
Generalized tonic-clonic (GTC) seizures are most strongly associated with
shoulder instability. Anterior instability is more common than posterior, as
the overall prevalence of anterior instability is greater.5 However, in
patients with posterior instability, the prevalence of seizure-related
instability is higher.

146. pt on antipsychotic medication, the greatest complication to happen:


A. Akathisia
B. Sedation

• Correct Answer:B
UpToDate:
Akathisia is less likely to occur in Second-Generation Antipsychotics
Sedation All Second-Generation Antipsychotics except pimavanserin are
histaminic H1 receptor antagonists and thus have potential to cause drowsiness.
As with other antihistamine-induced somnolence, the effect is most severe early in
treatment, and tolerance usually develops within a few days’ sedation/somnolence
related to alpha adrenergic antagonism.

147. 9 y male with history of headache for 5 days and neck stiffness for two days
With low grade fever for one month CSF analysis Turbid Glucose low WBCs neut:
27 Lymph: 87 Protein high Diagnosis:
A. bacterial meningitis
B. viral meningoencephalitis
C. TB meningitis

69
D. fungal meningitis

• Correct Answer:C
UpToDate:
Among patients with bacterial meningitis, the classic findings are:
●A CSF WBC count above 1000/microL, usually with a neutrophilic
predominance (TB has Lymphocytic Predominance)
●A CSF protein concentration above 250 mg/dL
●A CSF glucose concentration below 45 mg/dL (2.5 mmol/L)

148. patient had polyps excised from his sigmoid colon and on histopathological
report there was cancerous cells with free tissue margins, Next step:
A. Follow up
B. Sigmoidoscopy
C. Sigmoidectomy

• Correct Answer:A
UpToDate:
Polypectomy alone is adequate for polyps with high-grade dysplasia and Tis if the
resection margins are free of neoplastic tissue. Subsequent surveillance should be
performed at the same interval as for patients with high-risk adenomas

149- 14 y.o female with epigastric abdominal pain since a month Patient was
complaining that it is affecting her life, on exam patient had multiple bruises in
abdomen when asked about it She said it appeared after she fell over her cycle 18
days back, next inv?
A. CT abdominal
B. Abdominal US
D. laparotomy

• Correct Answer:A
UpToDate:
A history of any blow, crushing mechanism, or missile/implement trajectory to the
mid-to-upper abdomen should raise suspicion for pancreatic injury. Abdominal
tenderness and peritoneal signs on initial evaluation may indicate the presence of

70
intra-abdominal injury but are not specific for pancreatic injury. A physical exam
finding of abdominal wall ecchymosis may be a sign that a crush injury has
occurred. The imaging diagnosis of blunt pancreatic injury relies primarily on CT of
the abdomen.

150. Patient post RTA with massive bleeding in the nose and mouth with leg
fracture, decrease bp, Pt can take his breath, but he is afraid from what happened.
What is the next step?

A. Intubate with stretching cervix


B. Packing nasal bleeding
C. IV fluid

• Correct Answer: C
UpToDate:
The primary survey consists of the following steps:
●Airway assessment and protection (maintain cervical spine stabilization
when appropriate)
●Breathing and ventilation assessment (maintain adequate oxygenation)
●Circulation assessment (control hemorrhage and maintain adequate end-
organ perfusion)
●Disability assessment (perform basic neurologic evaluation)
●Exposure, with environmental control (undress patient and search
everywhere for possible injury, while preventing hypothermia)
-Cervical spine immobilization — Assume that an injury to the cervical spine
has occurred in all blunt trauma patients until proven otherwise.

151. Patient presented to ER after RTA, with SOB, in examination the tracheal
shifted to the left side, in chest X-ray, the lungs are expanded and winded
mediastinum. What is the diagnosis?
A. Tension pneumothorax
B. Cardiac tamponade
C. Pleural effusion
D. Ruptured esophagus

71
• Correct Answer: D (incomplete Q)
UpToDate:
thoracic and cervical radiography — Findings suggestive of an esophageal
perforation on chest radiograph include mediastinal or free peritoneal air or
subcutaneous emphysema. With cervical esophageal perforations, plain films of
the neck may show air in the soft tissues of the prevertebral space. Other findings
suggestive of an esophageal perforation include pleural effusions, mediastinal
widening, hydrothorax, hydropneumothorax, or subdiaphragmatic air.

152. Which of the following indicate the severity of asthma?


A. PEF less than 300 (Normal more than 400)
B. RR more than 55
C. PR 110

• Correct Answer: B
AMBOSS:
Severe Asthma Exacerbation Indices
Severe tachypnea (> 30/min)
Inspiratory and expiratory wheezing
Use of accessory muscles
Tachycardia (> 120/min)
Pulsus paradoxus
PEFR: < 40% predicted
SpO2 < 90%
Hypercapnia (PaCO2 ≥ 42 mm Hg

153. A 60-year-old man is admitted to the Coronary Care Unit with an acute
myocardial Infraction. His hemodynamic parameters 2 hours later are:
Blood pressure :80/50 mmHg Heart rate 40 /min Oxygen saturation °C 95% on
room air Which of the following would be the appropriate management?
A. IV0.6 mg atropine sulfate
B. Normal saline infusion
C. IV isoproterenol
D. IV dobutamine

72
• Correct Answer:A
UpToDate:
For Bradycardia patients with symptoms who have evidence of hemodynamic
instability, we administer atropine (1.0 mg intravenous [IV] push, which can be
repeated every three to five minutes, if needed, to a total dose of 3 mg)

154. Patient with intermenstrual bleeding. What is the most appropriate


investigation?
A) CBC
B) TFT
C) B-HCG
D)US

• Correct Answer:C (But Wafaa says D)


UpToDate:
Pregnancy test — Pregnancy should be excluded in all reproductive-age patients
with AUB. Pregnancy testing should be performed even in patients with recent
vaginal bleeding since this may represent bleeding during pregnancy rather than
menses. It should also be performed in patients who report no sexual activity and
in those who report use of contraception.
Role of ultrasound — While many patients will ultimately need an
ultrasound as part of the secondary evaluation, we typically avoid routinely
ordering it as part of the initial evaluation in all patients.

155. which vaccine can be given in immune deficient?


A. Dtap
B. MMR
C. Polio

• Correct Answer:A
UpToDate:
Live vaccines should be avoided
Viral — Live viral vaccines include:
●MMR (see "Measles, mumps, and rubella immunization in infants, children, and
adolescents")

73
●Measles-mumps-rubella-varicella
●Oral poliovirus (not available in the United States) (see "Poliovirus vaccination",
section on 'Live attenuated oral poliovirus vaccine')
●Live-attenuated influenza vaccine (see "Seasonal influenza in children: Prevention
with vaccines")
●Yellow fever (see "Immunizations for travel", section on 'Yellow fever vaccine')
●Varicella (see "Vaccination for the prevention of chickenpox (primary varicella
infection)")
●Herpes zoster (no longer available in the United States) (see "Vaccination for the
prevention of shingles (herpes zoster)")
●Rotavirus (see "Rotavirus vaccines for infants")
●Smallpox (vaccinia)
●Adenovirus (used predominantly in military personnel)
Bacterial — Live bacterial vaccines include:
●BCG (see "Vaccines for prevention of tuberculosis")
●Oral Ty21a Salmonella typhimurium

156. 35y.o man presented with fever, rigors & weight loss for 10 days. He
underwent Mitral valve replacement by prosthetic valve 1 month ago, Echo
showed a small vegetation. Vitals: Blood pressure 90/70 mmHg, HR 100/min, Temp
38.7°C. Caustive organisim is ?
A. Coxiella burnetli
B. Staphylococcus aureus
C. Streptococcus viridans
D. Staphylococcus epidermidis

• Correct Answer:B
During the initial two months of implantation, the most frequently
encountered pathogens were S. aureus and coagulase-negative
staphylococci (CoNS); next in frequency were gram-negative bacilli and
Candida species. This spectrum of organisms reflects the typical nosocomial
origin of these infections.
●Between 2 and 12 months after implantation, the most frequently
encountered pathogens were coagulase-negative staphylococci, S. aureus,
and streptococci, followed by enterococci. In general, cases occurring 2 to

74
12 months after surgery are a blend of delayed-onset nosocomial and
community-acquired infections.
●Beyond 12 months after implantation, the most frequently encountered
pathogens were streptococci and S. aureus, followed by coagulase-negative
staphylococci and enterococci. In general, the range of pathogens is similar
to that of native valve endocarditis (NVE) in patients who are not injection
drug users. This is because late PVE, like NVE, usually results from transient
bacteremia occurring among ambulatory patients.

157. Patient presented with sweating dyspnea, palpation and headache. She is a
known case of HTN and despite taking medication it’s not controlled. imaging
reveals supra renal mass. Which of the following meds u will give to control her
HTN?
A. CCB
B. Alpha blocker
C. ACEI
D. BB

• Correct Answer: B
AMBOSS:
Benign pheochromocytoma: primary therapy with phenoxybenzamine (non-
selective Alpha blocker)

158. Patient RTA with head trauma, increased urine output, decrease in Urine
osmolarity increased blood osmolarity?
A. Central diabetes insipidus
B. Cushing
C. Conn’s
D. Nephrogenic diabetes insipidus

• Correct Answer:A
UpToDate:

75
Central DI — Central diabetes insipidus (DI; also called neurohypophyseal or
neurogenic DI) is due to deficient secretion of ADH. This condition is most often
idiopathic (possibly due to autoimmune injury to the ADH-producing cells) or can
be induced by trauma, pituitary surgery, or hypoxic or ischemic encephalopathy.

159. Septic shock case indicates adequate systemic perfusion?


A-Cardiac index
B- Mixed venous oxygen saturation (SMVO2)
C-Central venous oxygen saturation (SCVO2)

• Correct Answer: B
UpToDate:
Hemodynamics – In addition, while dynamic measures of fluid responsiveness (eg,
respiratory changes in the radial artery pulse pressure) are preferred, static
measures of determining adequacy of fluid administration (eg, central venous
pressure 8 to 12 mmHg or central venous oxygen saturation ≥70 percent) may be
more readily available.

160. Patient 56 y.o came for routine follow up, known case of CHF asymptotic not
on medication his EF 25% what you want to give him?
A. Beta Blocker
B. ACEI
C. Spironolactone
D. Diuretics

• Correct Answer:B
AMBOSS:
• ACEI: For Every patient with HFrEF

76
161. DM, heart failure and admitted for hysterectomy and received Normal Saline
cause poor oral intake, 4 d nurse noticed decreases in Oxygen Sat. & SOB, o/e:
crackles, how to prevent this complication?
A. cardiopulmonary consult
B. daily fluid assessment
C. Order daily Lasix

• Correct Answer:B
UpToDate:
HF self-management — Effective care includes patient education and support to
promote HF self-management. HF-self management includes medication
management, daily monitoring of signs and symptoms (including daily weight
monitoring to detect fluid accumulation) and lifestyle modification

162. Patient k/c of COPD alert, has moderate respiratory distress.SO2 93%, ABG
showed hypoxia, hypercapnia and acidosis. What is next step?
A. noninvasive ventilation
B. Decrease oxygen
C. Increase oxygen
D. Mechanical ventilation

• Correct Answer:A
UpToDate:
Similar to at-home management, the major components of emergency
department or in-hospital management of exacerbations of COPD include
reversing airflow limitation with inhaled short-acting bronchodilators and systemic
glucocorticoids, treating infection, ensuring appropriate oxygenation, and averting
intubation and mechanical ventilation [1,23]. An approach to emergency
management of severe exacerbations of COPD is summarized in the table.

163. Typical case of hyperthyroidism and palpitation. What will you do for her
palpation?
A. PTU

77
B. Methimazole
C. Propranolol

• Correct Answer:C
AMBOSS:
reatment of hyperadrenergic symptoms: beta blockers (first line)

164. Patient with a fib and dilated cardiomyopathy, what to give to control the rate
now?
A. Digoxin
B. BB
C. Diltiazem

• Correct Answer:B
AMBOSS:
Pharmacological options for rate control- First-line
1-Beta blockers (e.g., metoprolol , atenolol , propranolol)
2- Nondihydropyridine calcium channel blockers (e.g., diltiazem, verapamil)
Avoid in patients with decompensated heart failure (LV systolic
dysfunction/low ejection fraction). Can be safely used in heart failure with
preserved normal LV systolic function.

165. Patient with TIA symptoms and Afib, what to give?


A. Aspirin
B. Warfarin INR goal 2-3
C. Warfarin INR goal 3-4
D. Enoxaparin

• Correct Answer:B
AMBOSS:Stable Afib of unknown duration or ≥ 48 hours: Preferably 3 weeks of oral
anticoagulation therapy with DOACs or warfarin (inr 2.5-3) before cardioversion.

78
166. A Patient had lower parathyroidectomy, came to the clinic with bone pain,
numbness around the lips. Lab results showed high Ca + high PTH. What’s the most
likely dx?
A. Missed adenoma
B. New adenoma
C. Parathyroid hyperplasia

• Correct Answer:A
UpToDate:
Missing gland — A missed parathyroid adenoma is the most common cause for
persistent hyperparathyroidism

167. Patient with raised JVP, Hypotension, unclear heart sound, clear lung sounds.
How to confirm the diagnosis?
A. ECHO
B. ECG
C. Chest X-ray
D. PFT

• Correct Answer:A
AMBOSS: Echocardiography
Indication: diagnosis and monitoring of all patients with suspected
pericardial effusion and/or tamponade
Procedure: TTE (gold standard) or focused-assessed transthoracic
echocardiography)

168. 43-year-old male with hypertension on -sartan & amlodipine, his blood
pressure is not controlled. What to add?
A. Lisinopril
B. Atenolol
C. Indapamide(thiazide)
D. Zocin (alpha blocker)

• Correct Answer:C

79
AMBOSS: First-line medications for primary hypertension are thiazide diuretics,
ACEIs, ARBs, and dihydropyridine CCBs.

169. Patient with HTN on 25 mg once daily hydrochlorothiazide. BP not controlled.


Management?
A. Double thiazide dose
B. Add ACEI
C. Add beta blocker

• Correct Answer:B
UpToDate: Another observation is that, with increasing the dose of a given
medication, the additional antihypertensive benefit diminishes while the risk of
adverse effects increases; as an example, the following findings were noted in a
2009 meta-analysis of randomized trials

170. How to know fetal weight intrapartum in 37w?


A. Femur length
B. head circumflex
C. Biparietal diameter
D. Abdominal circumference

• Correct Answer:D
UpToDate:
Estimating fetal weight – Fetal weight is estimated using formulas that incorporate
measurements of BPD, HC, AC, and/or FL. No formula is consistently superior and
the random error in fetal weight estimation exceeded 14 percent of birth weight in
5 percent of fetuses, in a systematic review.

171. elderly Patient with uncontrolled HTN, he is on ARBs, CCB, thiazide?


A. Add ACEI
B. Add BB
C. Add spironolactone

80
• Correct Answer:C
AMBOSS: Potassium sparing diuretics; Frequently used as add-on therapy in
resistant hypertension.

172. When does bedwetting becomes alarming?


A. 5 years
B. 6 years
C. 7 years
D. 8 years

• Correct Answer: C (Safdar and UTD)


AMBOSS: Diagnostic criteria for Nocturnal Enuresis
Occurs 2x/week for ≥ 3 months or causes clinical distress
Patient's developmental age must be ≥ 5 years

These skills usually are achieved, at least during the day, by approximately
four years of age. Nighttime bladder control is achieved months to years
after daytime control but is not expected until five to seven years of age

173. Thyroid cancer with increased calcitonin:


A. Medullary thyroid cancer
B. Follicular thyroid cancer

• Correct Answer:A
AMBOSS: Medullary carcinoma, Calcitonin: A hormone secreted by parafollicular
cells, which is the tissue of origin of medullary carcinoma

174. Female with previous 2 preterm labour, now she is in 20 weeks of gestation
and her cervix opened 30 mm, what you will do?
A- Immediate cervical cerclage
B- Give tocolytic & wait
C- Strict bed rest
D- progesterone supplement

81
• Correct Answer:D
AMBOSS: Cervical cerclage
Definition: placement of a supportive suture in the cervicovaginal junction
to prevent early pregnancy loss or preterm birth
Timing: < 24 weeks gestation; most commonly performed at 13–16 weeks
gestation
Indications: only in singleton pregnancies
Multiple previous preterm births or pregnancy losses in the second
trimester
A previous preterm birth and current ultrasound diagnosis of a shortened
cervix (cervix length < 25 mm) at < 24 weeks gestation
Cervical dilation on inspection at < 24 weeks gestation
Prior cerclage due to cervical insufficiency at < 24 weeks gestation
Contraindications
Preterm labor
Premature rupture of membranes
Chorioamnionitis or vaginal infection
≥ 24 weeks' gestation
Unexplained vaginal bleeding
Multiple gestations
Progesterone supplementation (vaginal or intramuscular): indicated for a
short cervical length at < 24 weeks' gestation in the absence of a previous
preterm birth
Strict bed rest is not recommended.

175. Normal vaginal delivery, Baby weight 4.2kg, Laceration which degree:
A. First
B. Second
C. Third
D. Forth

• Correct Answer:D
Wafaa ObGyn:

82
‫ لرصاحه ر يف‬ACOG + UpToDate: ‫كاتبي‬
‫ر‬ ‫ر‬
‫ كلهم‬third and fourth ،‫حاطي واحد لحاله‬ ‫مو‬
‫ر‬
‫ بس لقيت ف ابحاث وخرابيط جانبيه كاتبير‬fourth ‫)فـ رحت معهم (الموت مع الجماعه ارحم‬
‫ي‬

176. 41YO P5 +3 presented to the clinic complaining of abnormal uterine bleeding


her Menstrual period is regular, associated with blood clots and pain that is not
relieved by analgesic she had previous myomectomy, she is a known case of
PCOS&her BMI is 40?
A-Adenomyosis
B-Endometriosis
C-Uterine fibroid
D-Endometrial hyperplasia

• Correct Answer:A
Wafaa ObGyn: Adenomyosis risk factors:
⁃ Previous uterine surgery, C/S, D&C
⁃ Childbirth
⁃ Middle age

177.A 39-week pregnant patient, history of caesarean section, due to breech


presentations, now she is in labor, with regular contractions 4 every 5 min, cervix
fully dilated, full effacement, station +3, What is your management?
A. Ventouse delivery
B. caesarian section
C. examine her after 2 hours

• Correct Answer:A
Wafaa ObGyn According to ACOG: Similar standards should be used to evaluate
the labor progress of women undergoing Vaginal delivery after cesarean and those
who have not had a prior cesarean delivery.
Let’s Exclude!!
- Ventose Delivery -> because her cervix is fully dilated, and the head is
engaged +2 and beyond.
‫شء يخلينا نختار ال فينتوز مثل‬
‫ والسؤال ناقص الرصاحه اكيد فيه ي‬CTG category 2 or maternal
exhaustion or prolonged 2nd stage of labor
- Immediate CS -> if her CTG is category3

83
- Examine her after 2 hours -> if she wasn’t fully dilated and in active
pushing state!! ‫ساعتي رنبكها‬
‫ر‬ ‫ر‬
‫البر راسه بيطلع واالم الزم تدف خالص وش‬

if the choice was WAIT 2 more hours (NOT EXAMINE AFTER 2hrs) I would go
with it,
‫ بس هو‬،‫البين‬ ‫ ساعات واذا ماولدت نستخدم الفينتوز نسحب‬٣ ‫يعن المفروض انها تدف خالل‬‫ر‬
‫ي‬ ‫ي‬
‫الل كانت جالسه تدف فيه عشان أقول فينتوز وانا مرتاحه وبنفس‬
‫موضحل ال الوقت ي‬
‫ي‬ ‫بالسؤال مو‬
‫فيعن االصح هو فينتوز‬‫ر‬ ‫الوقت كل الخيارات الثانيه خطأ‬
‫ي‬

178. Patient with chronic limb ischemia, presented with sudden leg pain,
diminished popliteal and distal pulses in right leg, and diminished distal pulse with
intact popliteal in the left, what's the appropriate next action?
A. Heparin
B. CT angio
C. US
D. conventional angio

• Correct Answer: A
UpToDate: For those who present with acute limb ischemia, anticoagulation
typically with a heparin (bolus followed by infusion) and intravenous fluid therapy
should be immediately initiated prior to making plans for intervention. These can
be started while awaiting evaluation by a specialist trained to treat the embolic
event. Systemic anticoagulation should be initiated as soon as the diagnosis of
embolism is made on clinical grounds, and before subsequent imaging.

179. Elderly Patient with small lung nodule (<8 mm), he is asymptomatic, not
smoker, no family hx of cancer, no constitutional, your next step?

A. f/u with CT scan


B.Biopsy
C.Chemotherapy

• Correct Answer:A
UpToDate: nodules ≤8 mm (without documented growth) are often followed with
serial CT because these lesions have a low likelihood of malignancy such that the
benefits of resection do not justify the risk of surgery.

84
180. 65-year-old Female Patient with AF, hx of Dm, HTN, TIA, calculate her
CHA2DS2- VASc score?
A. 3
B. 4
C. 5
D. 6

• Correct Answer:D
UpToDate:
Cha3ds2 criteria

Sex

Female (1 point)
Male (0 points)
Age

≤64 years old (0 points)


65 to 74 years old (1 point)
≥75 years old (2 points)

Comorbidities

Heart failure (1 point)


Hypertension (1 point)
Diabetes mellitus (1 point)
History of stroke, TIA, or thromboembolism (2 points)
Vascular disease (history of MI, PAD, or aortic atherosclerosis) (1 point)

181. Patient with PAD has 100-meter claudication's, DM, heavy smoker, not getting
better. How to improve his walking distance?
A. Supervised exercise program
B. Strict glycemic control

85
C. Smoking cessation

• Correct Answer:A
UpToDate: Improvement in walking capacity is generally better for supervised
compared with non-supervised exercise therapy, and exercise therapy compared
with available pharmacologic therapies.

182. Elderly pt moved to care center before 4 months, now he c/o loss of wt &
appetite, low mood and loss of interest for 4 w, also complain of loss of short
memory with attacks of low mood, what is the Dx
A. Alzehaimer
B. Depression
C. Parkinsonism

• Correct Answer:B
UpToDate:
Depressive symptoms and their context — The interview should:
●Establish the presence of depressive symptoms:
•Depressed mood most of the day
•Loss of interest or pleasure in most or all activities
•Insomnia or hypersomnia
•Significant weight loss or weight gain (eg, 5 percent within a month) or
decrease or increase in appetite nearly every day
•Psychomotor retardation or agitation nearly every day that is observable
by others
•Fatigue or low energy
•Decreased ability to concentrate, think, or make decisions
•Thoughts of worthlessness or excessive or inappropriate guilt
•Recurrent thoughts of death or suicidal ideation, or a suicide attempt

183. SLE Patient on hydroxychloroquine and Mycophenolate mofetil, what


is the best non- pharmacological management?
A. Smoking cessation
B. Weight reduction

86
• Correct Answer:A
UpToDate: Diet and nutrition — Limited data exist concerning the effect of dietary
modification in SLE.

Smoking cessation — Patients should be counseled against cigarette smoking,


since it has been associated with more active disease [65,66]. Smoking adds to the
baseline increased risk of accelerated atherosclerosis with coronary heart disease
in those with SLE. There is also evidence to suggest that smoking diminishes the
efficacy of hydroxychloroquine.

184. 30s female complaining of SOB arthralgia dyspnea fever she had dental
procedure 2 weeks ago and another procedure 2 months ago on PE there is heart
murmur radiation to axilla.UA showed hematuria and proteinuria what is the
diagnosis?
A. Infective endocarditis
B. Post streptococcus Glomerulonephritis
C. SLE

• Correct Answer:A
AMBOSS: Modified Duke criteria:
Major
1-Positive blood cultures (any of the following):
Typical organisms from two separate samples
Viridans streptococci S. aureus S. Gallolyticus HACEK group
Community-acquired enterococci without a primary focus
Persistently positive cultures with microorganisms consistent with endocarditis
≥ 2 positive blood cultures drawn > 12 hours apart
All of 3 or a majority of ≥ 4 separate blood cultures
One positive culture for Coxiella burnetii or an anti-phase 1 IgG titer of ≥ 1:800
2-Characteristic echocardiographic findings of IE or New valvular regurgitation
Minor:
1-Predisposing condition (e.g., underlying heart abnormality, IV drug use)
2-Fever > 38°C (100.4°F)
3-Vascular abnormalities
4-Immunologic phenomena

87
5-Microbiology: positive blood cultures not fulfilling major criteria
Diagnostic category
≥ 2 major criteria ≥ or 1 major criterion PLUS ≥ 3 minor criteria or ≥ 5 minor criteria

185. patient has midsystolic murmur grade 2/6 best heard at right lower sternal
border, and has S4 NO S3 best heard at the apex. What is the most likely
diagnosis?
A. Aortic stenosis
B. Mitral regurgitation
C. Aortic regurgitation
D. Mitral stenosis

• Correct Answer:A
AMBOSS:

AS: Harsh crescendo-decrescendo (diamond-shaped), late systolic ejection murmur


that radiates bilaterally to the carotids
Best heard in the 2nd right intercostal space

186. which of these indicate severe asthma?


A. PEF <250
B. RR >20
C. HR 100/min
D. O2sat 95%

• Correct Answer:A
AMBOSS: Severe Asthma Exacerbation Indices
Severe tachypnea (> 30/min)
Inspiratory and expiratory wheezing
Use of accessory muscles
Tachycardia (> 120/min)
Pulsus paradoxus
PEFR: < 40% predicted
SpO2 < 90%

88
Hypercapnia (PaCO2 ≥ 42 mm Hg

187. Patient K/C of SLE, which of the flowing type of anemia associated with SLE?
Hemoglobin electrophoresis: normal, Ferritin: High, Reticulocyte: High, Lactate
dehydrogenase: high, Haptoglobin: low?
A. Anemia of chronic disease B. Hemolytic anemia
C. Iron deficiency
D. Thalassemia

• Correct Answer:B
UpToDate:
Lab Findings for hemolytic anemia
Anemia that is not due to another obvious cause.
●Increased reticulocyte count that is not explained by accelerated RBC
production due to recent bleeding; repletion of iron, vitamin B12, folate, or
copper; or administration of erythropoietin.
●Signs of RBC destruction such as increased lactate dehydrogenase (LDH),
low haptoglobin, increased unconjugated bilirubin.

188. which GFR metformin is harmful for diabetic pt ?


A. 30-40
B . <30
C . <15

• Correct Answer:B
UpToDate: Metformin is contraindicated in patients with factors predisposing to
lactic acidosis. These predisposing factors/contraindications are:
Impaired kidney function (estimated glomerular filtration rate [eGFR] <30
mL/min/1.73 m2)
Concurrent active or progressive severe liver disease
Active alcohol abuse
Unstable or acute heart failure at risk of hypoperfusion

89
Past history of lactic acidosis during metformin therapy
Decreased tissue perfusion or hemodynamic instability due to infection or other
causes

189. At what age baby talk 6-10 word and recognize two body part
A. 17 month
B. 19month
C. 24 month
D. 12month

• Correct Answer: B not sure


Dr. Safdar: 18 months (6 to 10 words)

191. Child 4 years old with asymptomatic umbilical hernia what's the treatment?
A. Reassurance
B. wait until 5 years
C. Surgical repair

• Correct Answer:B
AMBOSS:
Conservative: ∼ 90% will spontaneously close by 2 years of age
Surgery (rarely necessary)
Large umbilical hernias (defect > 1.5–2 cm) in children > 2–3 years of age
No evidence of spontaneous closure by 5 years of age
Patients with incarcerated, obstructed, or strangulated umbilical hernias

192. A female brought her baby to the well baby clinic ,he is still on formula /breast
fed the doctor advised her to give the baby iron supplement after the appointment
How old is the infant now?
A- 1 month
B- 2 months
C- 3 months
D- 6 months

90
• Correct Answer:D
UpToDate:
After six months of age, the volume of human milk ingested by exclusively
breastfed infants generally becomes insufficient to meet the infant's requirements
for energy, protein, iron, zinc, and some fat-soluble vitamins

193. 4 years. old girl with decrease head growth, decrease social interaction,
decrease in language , what is the closest ddx?

A. Autisim
B. mental retardation
C. Rett's disorder
D. aSPERGER syndrome

• Correct Answer:C

194. Patient “45 years old, came with history reducible hernia in inguinal area it
pop out every two days ”interval” with mild pain and and he Manuel reducible it by
his finger, then he suddenly developed severe “constant ” pain in early morning in
that area and came to hospital, on examination of the inguinal area and scrotum
examination and he is free from both them and pain is disappear Now just with
persistent nausea and vomiting next step ?
A. Pelvic and Abdomen CT
B. hernia repair today after 2 hour i think C reasurance and discharger

• Correct Answer:A
AMBOSS:Indications for imaging:
Uncertain clinical diagnosis
Suspected recurrent or occult inguinal hernia
Consider preoperative imaging for complicated inguinal hernia.
Modalities
First line: ultrasound of the groin
Inconclusive ultrasound findings: CT or MRI abdomen and pelvis
Complicated inguinal hernia: Consider CT abdomen and pelvis with IV contrast.

91
Suspected occult inguinal hernia or recurrent inguinal hernia; consider any of the
following modalities: MRI groin, Dynamic CT or dynamic MRI, if
available Herniography

195. female with Pelvic pain increases with menses. On examination: uterus
normal And there is uterosacral nodularity and tenderness dx?
A. Endometriosis
B. Adenomyosis
C. PID
D. Fibroids

• Correct Answer:A
An article on pubmed:
Uterosacral nodularity was pathognomonic of endometriosis in infertile women.

196. 34 years female at 28 weeks with cough and difficulty in breathing. She
appeared restless and uncomfortable. Auscultation of the lung shows bilateral
rhonchi, the nail bed and oral mucous membrane appear pale, she is allergic to
dust, pollen, mold and animal hair. No hx of smoking. She reports that the
symptoms began 2 days prior and started cough with clear color phlegm. All vital
normal. What it's the most appropriate initial test?
A.Chest-Xray
B.Hgh
C.Spirometry
D.ABG

• Correct Answer:D
Master the boards: The best initial test in an acute exacerbation: peak expiratory
flow (PEF) or arterial blood gas (ABG). Peak flow can be used by the patient to
determine function.
Chest X-ray is most often normal in asthma, but may show hyperinflation.
Chest x-ray is used to: Exclude pneumonia as a cause of exacerbation, Exclude
other diseases such as pneumothorax or CHF in cases that are not clear

92
197. Mother brought her 9 years old child complainiong of short stature. Physical
exam was normal, GH normal, FSH normal, TSH normal, T3-4normal, and insulin
like growth factor decreased. X ray of the bone shows an age of 6 years. What is
the diagnosis ?
A. Familial short stature
B. GH deficiency
C. Constitutional growth delay

• Correct Answer:B
Safder’s slides: GH deficiency has the bone age delayed by more than 2 years and
the IGF-1 is low.

198. Pediatric age pt presented with his mom came to the ED. She said that her
child ingested large amount of paracetamol. The pt was healthy with no signs of
any toxicity on presentation. They observed the pt for 4 hours with no change in
health, no signs and symptoms of toxicity. The mom then mentioned that she
found some of the drug was spilled on his clothes and in the floor, what is the next
step?
A. Reassure.
B. Watch for another 4 hours.
C. Give antidote.

• Correct Answer: C
AMBOSS: Treatment involves administration of activated charcoal and N-
acetylcysteine. Liver transplant may be necessary. Management includes
• Activated charcoal administered < 4 hours after ingestion
• Measure acetaminophen (APAP) levels 4 hours after ingestion (or
immediately, if ingestion occurred > 4 hours prior to presentation)
• Antidote: PO or IV N-acetylcysteine (NAC)
• Administered 4–24 hours after ingestion
• NAC replenishes glutathione stores in the liver
• Treatment of liver failure
• Liver transplant in severe cases

93
199. Which imbalance is responsible for violence?
A. Low serotonin.
B. Low testosterone.

• Correct Answer: A
NCBI: Testosterone activates the subcortical areas of the brain to produce
aggression, while cortisol and serotonin act antagonistically with testosterone to
reduce its effects.

200. 62 years old pt known to be diabetic complaining of cough, SOB, and fever.
Chest exam reveals crackle. Small pleural effusion is present. Dx is pneumonia.
Which of the following is poor prognosis factor?
A. Age
B. DM
C. Crackles
D. Pleural effusion

• Correct Answer: A
UpToDate: Patient characteristics associated with increased mortality include older
age and the presence of multiple chronic comorbidities. Most deaths (>60 percent)
occurred in those ≥65 years old and in those with two or more chronic
comorbidities

201. Elderly was completely healthy except for elevated BP for the first time. What
should you do?
A. Ambulatory BP measurement.
B. Start anti HTN
C. Measure the BP two times later in the clinic
D. Measure BP two time in home

• Correct Answer: D
UpToDate: In addition to obtaining multiple blood pressure measurements, blood
pressure should be measured in both arms, at least at the initial visit. In older
individuals or those with potential orthostatic symptoms, postural measurements
should also be taken.

94
202. A 45-year-old smoker came to the clinic for his diabetes follow-up. During
discussion, he acknowledged that smoking is not good for his health. He plans to
quit this year. According to the Stages of Change Model, at what stage of change is
this patient?
A. Precontemplation
B. Contemplation
C. Preparation
D. Maintenance

• Correct Answer: C
First Aid for the USMLE Step 1: As stated in the question, he PLANS to quit
smoking.

203. Case of Asthma exacerbation. Patient presents with SOB, cyanosis , unable to
complete sentence. Vitals = (Low O2 maybe) / RR : Not sure. ABG: respiratory
acidosis. What is the appropriate next step?
A. IV mg sulfate
B. Intubation
C. Non invasive mechanical ventilation

• Correct Answer: B
(can’t complete sentences, unstable vital signs indicate severe).
AMBOSS: Severe Asthma Exacerbation Indices:
Severe tachypnea (> 30/min)
Inspiratory and expiratory wheezing
Use of accessory muscles
Tachycardia (> 120/min)
Pulsus paradoxus
PEFR: < 40% predicted
SpO2 < 90%
Hypercapnia (PaCO2 ≥ 42 mm Hg

95
204. When should you introduce solid food to children?
A. At 6 months.
B. At 3 months.
C. At 8 months.
D. At 7 months

• Correct Answer: A
Dynamed: Solid foods and nonmilk fluids, in addition to breast milk or formula, can
be introduced at age 6 months and in a way that maximizes nutrient requirements
and minimizes adverse reactions and the development of allergies.

206. A pregnant lady comes for a regular checkup. Labs shows anemia with high
MCV. A blood smear is shown below. What is the most likely diagnosis?
A. Folic acid deficiency
B. 12 deficiency
C. IDA
D.Thalassemia minor

• Correct Answer: A (Picture shows hyper-segmented neutrophils)


UpToDate: Folate deficiency is the most common cause of megaloblastic anemia
during pregnancy

207. Child presents with unilateral wheeze. What is the most likely diagnosis?
A.Foreign body aspiration
B.Asthma

• Correct Answer: A
AMBOSS: Bronchial asthma presents with bilateral wheezing, as opposed to the
unilateral wheeze seen in foreign body aspiration.

208. 3-month-old coming with staccato cough (meaning machine-gun like cough),
history of conjunctivitis is noted. No history of diarrhea. Has basal crackles on
auscultation?
A. Chlamydia pneumonia
B. Adenovirus pneumonia
C. Pertussis

96
D. Mycoplasma pneumonia

• Correct Answer: A

209. 40 or 50 female k/c of dm Came with retrosternal pain at rest for 3 hours.
ECG was done showing a T inversion in (V2 V5). Labs: high troponin 10. What is
your diagnosis?
A. Stable angina
B. Unstable angina
C. St elevation infraction
D. Non St elevation infraction

• Correct Answer: D
According to AMBOSS:
ECG findings:
No ST elevations.
Normal or nonspecific (e.g., ST depression, loss of R wave, or T-wave inversion).

210. Pt Dx grand mall seizure from childhood on NA valproate develop. Pt c/o


generalized tonic colonic seizure which drug IV use in management???
A.Phenytion
B.Diazepam
C.Phenobarbitone

• Correct Answer: B
UpToDate: Status epilepticus treatment:
1st: benzodiazepines
2nd: IV (phenytoin, valproate or levetiracetam)
3rd: ICU + IV (phenobarbital, midazolam or Propofol)

211. 8year old diabetic female with dysuria. Urine culture showed 10,000 E.coli .
High wbc and high creatinine. Which abx is contraindicated?
A.Nitro
B.Ceftriaxone

97
C.Trimethoprim-sulfamethoxazole

• Correct Answer: A
AMBOSS: nitrofurantoin contraindications
Children < 1 month of age
Breastfeeding women
Women at 38–42 weeks' gestation or during delivery
Hepatic dysfunction [70]
Renal dysfunction with a creatinine clearance < 60 mL/min

212. Pt with painless defecation and blood in the stool. Mass prolapse with
straining but is reduced spontaneously when resting in 3 areas 3.7....Hb very low.
Dx hemorrhoids was given. Which of the following is Definitive management?
a. Conservative management
b.Fiber diet
c. Rubber band ligation
d.Hemorrhoidectomy

• Correct Answer: C
AMBOSS: if the question ask about the initial, start with non-reactional measures.
Definitive for class 2 hemorrhoids is rubber band ligation. Hemorrhoidectomy is
reserved for class 3 and 4.

213. Pt received 3 dose of HB vaccine came to clinic


a. HBsAng
b.HBeAg
c. HBsAB +ve
d.HBcAB+HBsAB +ve

• Correct Answer: C
UpToDate:

98
214. Pregnant with candida? How will you treat the patient?
A.Oral anti fungal
B.Topical anti fungal
C.Oral abx

• Correct Answer:B

99
UpToDate: For pregnant persons, we suggest a topical imidazole
(clotrimazole, miconazole) vaginally for seven days rather than a nystatin
pessary or an oral azole (Grade 2C). Case reports have described a pattern
of birth defects (abnormalities of cranium, face, bones, and heart) after first
trimester exposure to high-dose oral azole therapy (400 to 800 mg/day) and
cohort studies have reported conflicting data on risk of miscarriage.

215.56 Y.0 male patient presented with the classical triad of polydipsia, •
polyphagia and polyuria. A diagnosis of Diabetes Mellitus was made. Which of the
following causes impaired wound healing in diabetic patients?
a. Decreased blood supply
b.Impaired phagocytosis
c. Autonomic neuropathy
d.Dehydration

• Correct Answer: B (Um Alqura)


UpToDate: Numerous cytologic factors contribute to impaired wound
healing in patients with diabetes. These include decreased or impaired
growth factor production, angiogenic response, macrophage function,
collagen accumulation...

Another recall: A diabetic patient presented with exudates from a wound in


his leg with poor healing and no sign of inflammation. What is the cause of
the poor wound healing in this case?
A.Decreased phagocytic process
B.Stimulated bacterial growth
C.Deacred immunity
D.Increased blood supply
Correct Answer: A

216. 71 year old diabetic patient with pneumonia came with


parapneumonic effusion what will make the prognosis poor?
a. DM
b.High WBCs

100
c. Pleural effusion
d.Age

• Correct Answer: D
UpToDate: Key factors that predict short-term mortality include severity of
illness, older age, and the presence of morbidities. Most deaths (>60
percent) occurred in those ≥65 years old and in those with two or more
chronic comorbidities.
Dr. Jalilah: If age more than 65 yrs it will be age.
If less than that it will be DM.

217. ‫ صوره‬US CBD ‫سيناريو‬


Patient c/o RUQ Pain with N/V tender examination. Investigation . High liver
enzymes . High Bilirubin High TWBc US.. Show GB 1.5cm stone CBD
dialation 12 mm. Most appropriate management?
a. lap cholecystectomy
b.CT scan
c. ERCP

• Correct Answer: C
UpToDate: For all other patients at high-risk for having common bile duct
(CBD) stones, options include ERCP with stone removal followed by elective
cholecystectomy or cholecystectomy with intraoperative common duct
exploration/postoperative ERCP. For most patients, the choice of treatment
depends on available expertise and patient preference.

218. Pregnant lady 30 wk with lethargy and fatigue tremor i think c/o neck
swelling Investigation TST. 0.1 ONLY US diffuse multi nodular swelling Most
appropriate management?
A. FNA
B. RA iodine
C. Thyroid surgery
D. Anti-thyroid drug

101
• Correct Answer: D
UpToDate: Thionamides are the primary treatment of hyperthyroidism due
to Graves' disease, toxic adenoma, or toxic multinodular goiter during
pregnancy.
Dr.Jalilah: In 1st trimester =PTU, 2nd trimester and more carbimazole.

219. 45 y/o female complete mammogram screening when to do it again ?


a. after 1 year
b.after 2 years
c. no need
d.forgot but i think after 6 months

• Correct Answer: A
American Cancer society: 45–49 years: should start annual mammography
screening.

220. Aggressive behaviour, what is the cause ?


a. low serotonin
b. high serotonin
c. low endorphin
d. high endorphin

• Correct Answer: A
PubMed: These results were interpreted to suggest that
a deficiency in serotonin was largely responsible for these men's aggressive
behavior.

221. 19 y male with history of headach for 5 days and neck stiffness for two
days With low grade fever for one month CSF analysis Turbid Glucose
normal range WBCs neut: 27 Lymph: 87 Protein not sure Diagnosis:
a. bacterial meningitis

102
b. viral meningioencephlitis
c. TB meningitis
d. fungal meningitis

• Correct Answer: B
AMBOSS:

222. Normal pregnant women at 12 weeks, Indication of sever preeclampsia


a. Increase cr
b. Increase urea
c. Increase Na level

103
• Correct Answer: A
UpToDate: Serum creatinine >1.1 mg/dL (97.2 micromol/L) or doubling of
the creatinine concentration in the absence of other renal disease.

223. Lady 29 week pregnant Came with bleeding and Vesicle Structure
came out what is your Dx ?
A. partial mole
B. complete mole
C. threatened abortion
D. missed abortion

• Correct Answer: B
AMBOSS: passing of Vesicles that may resemble a bunch of grapes

224. RA case on Ibuprofen and Methotrexate for 6 months, came with


exacerbations. What to add?
a. Adalimumab
B.Azathioprine
C.Cyclophosphamide
D. Hydroxychloroquine

• Correct Answer: A

104
UpToDate: In patients resistant to initial DMARD therapy (eg, MTX), we
usually treat with a combination of DMARDs (eg, MTX plus SSZ and HCQ, or
MTX plus a TNF inhibitor), while also treating the active inflammation with
antiinflammatory drug therapy.

225.A female presented with back pain and fever, she was given NSAIDs and
advised for bed rest at home, then she presented with inability to move her
lower limbs, O/E there was tenderness on the midback. Spinal MRI: Diffuse
densities on T6, What is the most appropriate next step?
A.Reassurance
B. Specimen from T6 vertebra
C. Brucella titer
D.Bone marrow aspiration for culture

• Correct Answer: C
UpToDate: Spondylitis is a serious complication of brucellosis; it is more
prevalent in older patients and patients with prolonged illness prior to
treatment. The lumbar vertebrae are involved more frequently than the
thoracic and cervical vertebrae, and associated paravertebral, epidural, and
psoas abscesses have been described, For patients with suspected
brucellosis, blood cultures and serologic testing should be performed. In
addition, laboratory studies including complete blood count and liver
function tests should be obtained.

226. Patient with HCV. He developed abdominal pain. A CT was done and
showed HCC which is 6cm. How do you treat this patient?
a. Radiotherapy.
b.Chemotherapy.
c. Excision

• Correct Answer: C

105
227. Postmenopausal woman complaining of 1 year hx of recurrent vulvar
itching associated with blood streak secretions. She recently develop pea
size mass in the labia/ What is the most likely diagnosis?
a. Bartholin cyst.
b.Cystic adenosis.
c. Bartholin gland cancer.
d.Squamous cell cancer of vulva

• Correct Answer: D
postmenopausal state, makes malignancies the more reasonable answers .

228. Female 27 years old, she is asymptomatic, her last pap smear was 3
years ago, and it showed unconcerned squamous cells. What is the most
appropriate thing to do?
a. Repeat pap with cytology.
b. No need and reassure.
c. Colposcopy.
d. Cervical swab

• Correct Answer: A
Dr. Wafaa:

229. Female patient came to the clinic complaining of a mass on a vagina.


She has a history of repeated unprotected intercourse with multiple
partners. Upon examination, she has a wart in the vagina, the causative
agent is:
a. Herpes simples. (usually multiple)
b.Neisseria Gonorrhoea
c. Treponemma pallidum
d.Molluscum contagiosum (benign self limiting)

• Correct Answer: Most likely C.

106
230. A case of PSGN. What is the highest confirmatory test?
a. Serum albumin
b. Proteinuria
c. Kidney US

• Correct Answer: B
AMBOSS: PSGN typically manifests as nephritic syndrome with hematuria,
mild proteinuria, edema, and hypertension.
Serum albumin not done in PSGN and Kidney US not routinely done.

231. Which valvular heart disease has the worst outcome in pregnancy?
a. Mitral regurgitation
b. Mitral stenosis
c. Aortic stenosis
d. Aortic Regurgitation

• Correct Answer: B
American Cardiac Society
Mitral Stenosis:
Moderate to severe mitral stenosis (MS) is poorly tolerated in pregnancy. As
the heart rate increases, diastolic filling time is decreased, which leads to
increased pulmonary hypertension and pulmonary venous congestion. As a
result, women with moderate to severe MS who were asymptomatic prior
to pregnancy may become symptomatic during pregnancy. Arrhythmias,
specifically atrial fibrillation (AF) and supraventricular tachycardia, are
increased and occur in as many as 11% in one study of women with
rheumatic mitral valve disease. Poor fetal outcomes including fetal growth
restriction, low birth weight and preterm birth increase with increasing
severity of MS.

232. Patient hear aliens ‫ يسمع أصوات كائنات فضائيه تتكلم معه‬Dx.

107
a. Anxiety disorder
b.Severe depression
c. bereavement disorder
d.severe bipolar +not remember

• Correct Answer: D (by exclusion and most others say that the thing they
don’t remember is Psychosis which is a key word).
AMBOSS:
Psychosis: an impaired perception of reality evidenced by one or more of the
following thought disturbances
Hallucinations
Delusions
Disorganized thinking, speech, or behavior
Primary psychosis: psychosis resulting primarily from a psychiatric disorder, e.g.,
schizophrenia .
Secondary psychosis: psychosis resulting primarily from a general medical
condition and/or the effect of a substance
Psychotic disorder: a disease or condition that produces psychosis

233. Female pt with 2 months with jaundice fatigue and pruiritis. Obese
Medically free High AST ALT ALP. Viral serology negative. +ANA CT Liver
Plasma cell Management?
a. Vit E and weigh loss
b.Methotrexate
c. Steroid

• Correct Answer: C
autoimmune hepatitis Our diagnostic criteria — The diagnosis of autoimmune
hepatitis can be made in a patient with a compatible clinical presentation when the
following features are present (see 'Patterns of clinical presentation' above):

●A minimum of one elevated serum aminotransferase, typically (but not always)


an aspartate aminotransferase (AST) and/or alanine aminotransferase (ALT) level
at least two times the upper limit of the reference range.

108
●A minimum of one positive laboratory test: increased total IgG or gamma-
globulin levels, and/or serologic markers (antinuclear antibodies [ANA], anti-
smooth muscle antibodies [ASMA] at a titer of at least 1:40, anti-liver/kidney
microsomal-1 [anti-LKM-1] antibodies, anti-liver cytosol antibody-1 [ALC-1], or anti-
soluble liver/liver pancreas [anti-SLA/LP] antibodies).

●Exclusion of diseases that have a similar presentation, particularly viral hepatitis,


drug-induced liver injury, and alcoholic liver disease. (See 'Differential diagnosis'
below.)

When a liver biopsy is obtained, the diagnosis can be confirmed by histology


showing interface hepatitis and/or a predominantly lymphoplasmacytic infiltrate.
We prefer prednisone monotherapy (eg, 40 to 60 mg daily) for the initial
treatment of most patients with autoimmune hepatitis

234. Pregnant unbooked presented with vaginal bleeding no abdominal


pain, fundal height 34. have no transportation method to hospital. NEXT
step:
A. US
B. Admit to ward
C. Cervical exam

• Correct Answer: B
Amboss: Therefore, it is likely that most of these patients can be managed on an
outpatient basis until vaginal bleeding occurs or until admission for scheduled
cesarean birth. However, patient-specific risk factors need to be taken into
account. Such factors may include:
●Short cervical length on transvaginal ultrasound examination (eg, there is an
increased risk for preterm labor if cervical length is ≤25 mm, and we consider
hospitalization if ≤15 mm)
●Rapid cervical shortening (eg, >10 mm over a one- to two-week period on
transvaginal ultrasound)
●Inability to get to the hospital promptly (within approximately 20 minutes),
and lack of home support in case of an emergency

Dr. Wafaa:

109
Any woman with placenta previa can’t be discharged from the hospital unless she
is able to return to the hospital within 20 minutes.

235. Patient known hypertensive on lisinopril and amlodipine, recently


diagnosed as TB What you will do?
a. Stop lisinopril
b.Stop amlodipine
c. Increase lisinopril
d.Stop rifampicin

• Correct Answer: C
AMBOSS: (Explanation written for amlodipine but may also affect ACEi but most
resources say its minimally affected at best)
Calcium-channel blocking agents (diltiazem, nifedipine, verapamil)
Potential increased metabolism of the calcium-channel blocking agent [ref]
Decreased plasma verapamil concentrations; loss of therapeutic effect of oral
verapamil reported
Dosage adjustment of the calcium-channel blocking agent may be needed

236. patient in 40 came to with fibro adenoma of breast and you did
excision and in the lab, they till you there its benign and inside it there is
cancerous O/E no lymph node palpable in axilla and everything is normal
what will you do for her?
a. Mastectomy
b.follow up

• Correct Answer: B
AMBOSS:
Treatment by stage
Stage 0: DCIS
Surgery
Most commonly BCT, potentially including SNLB
Simple mastectomy when DCIS is too large for BCT, potentially including SNLB

110
Radiation therapy: following BCT
Systemic therapy: adjuvant hormone therapy if DCIS is hormone receptor-
positive
Stage I
Surgery: BCT or mastectomy, in combination with either SNLB or axillary
lymph node dissection
Radiation therapy: following BCT
Systemic therapy
Adjuvant hormone therapy for hormone receptor-positive breast cancer
Adjuvant chemotherapy for tumors larger than 1 cm in diameter and/or
breast cancer that is hormone receptor-negative/HER2-positive
Adjuvant targeted therapy for HER2-positive breast cancer
Stage II
Surgery: BCT or mastectomy, in combination with either SNLB or axillary
lymph node dissection
Radiation therapy
Following BCT
After mastectomy, when lymph node status is positive, or margins are not
tumor-free after mastectomy
Systemic therapy: Depending on the individual case, hormone therapy,
chemotherapy, and targeted therapy can be either neoadjuvant, adjuvant,
or both.
Stage III
Neoadjuvant approach
In most cases, treatment of stage III begins with neoadjuvant
chemotherapy, potentially in combination with targeted therapy for HER2-
positive breast cancer to shrink the tumor.
Afterwards, either BCT or mastectomy is performed, usually in combination
with ALND.
This is usually followed by radiation and, in some cases, chemotherapy
and/or targeted therapy.
Individuals with hormone receptor-positive breast cancer will receive
adjuvant hormone therapy.
Surgical approach
In most cases, mastectomy for locally advanced cancer, together with ALND
Surgery is followed by radiation, adjuvant chemotherapy, and, depending
on receptor status, hormone therapy and/or targeted therapy.

111
Stage IV
Palliative therapy in stage IV consists of systemic treatment (chemotherapy,
hormone therapy, targeted therapy, and possibly immunotherapy), possibly
in combination with radiation therapy.
Additionally, drugs to relieve symptoms of metastatic disease can be given
(e.g., bisphosphonates, pain medication, antiemetics).
In some cases, palliative surgery can be done to control local symptoms
(e.g., mastectomy).
Special patient groups: gestational breast cancer
Surgery is the treatment of choice.
Radiation therapy is contraindicated during pregnancy.
Adjuvant chemotherapy only in the second and third trimester

237. 12-year old boy injured with supracondylar fracture and distal radial
pulse absent, which of the following is the appropriate next step in
management?
A-K wire
B-intramedullary nail
C-surgical exploration
D-Closed reduction

• Correct Answer: D
Current + UpToDate: The radial pulse and the function of the radial, ulnar,
and median (including anterior interosseous) nerves should be checked
immediately. If ischemia is present initially, prompt reduction of deformity is
the next step toward restoration of normal perfusion.

238. 70 y old male K/O HTN, DM, hyperlipidemia on madication ( lesinopril,


amlodipine, statin) came to reguler check and by examination you find
congestive heart with bilaterly lung cripitation and mild LL edema. BB 155/
85, HR 110, Spo2 94%. What drug to add to his medication?
A) spironolactone
B) frusemide
C) doxazosin
D) atenolol

112
• Correct Answer: B
AMBOSS:

239. Child with Sx of tracheomalacia, How to confirm the diagnosis?


A) chest XR
B) bronchoscopy
C) Fluoroscopy

• Correct Answer: B
Bronchoscopic visualization of dynamic airway collapse (dynamic flexible
bronchoscopy (DFB)) is considered by many experts the diagnostic gold
standard.

240. Patient diabetic admitted to hospital on Metformin , which is best


during hospitalisation :
A) Metformin
B) Sliding scale insulin
C) Pre and post prandial insulin

• Correct Answer: B
AMBOSS: + UpToDate: A basal-bolus insulin regimen is preferable but we can use
Sliding scale insulin but Prolonged usage is discouraged

113
241. Stab wound to the neck and injury to Zone 3. There was active
bleeding. They did not mention if stable or not. There was no Vitals. What is
the most proper initial management?
A. Open repair
B. Endovascular repair
C. Artery ligation

• Correct Answer: B
UpToDate: Penetrating neck injuries in zone III are very difficult to access
surgically. Typically, only bleeding injuries are likely to require emergent/urgent
management. Such vascular injuries are best managed via an endovascular
approach

242. A 54 yo female medically free comes for routine checkup ,Her cardiac
exam revealed grade 4 pansystolic murmur heard best at the apex and
radiated to the axilla, she is asymptomatic and the rest of her exam is
normal, best next step to confirm the diagnosis?
A. TTE
B. TEE
C. ASO TITER
D. CHEST X ray

• Correct Answer: A
UpToDate: The diagnosis in patients with suspected MR is generally confirmed by
transthoracic echocardiogram (TTE)

243. Man came with scalp open wound, after 6h assault, what wound
management?
A-secondary closure
B-Debridment with 1ry closure
C-Debridment with granulation
D-Leave it for granulation! I think wrote like this

114
• Correct Answer: B
AMBOSS: Debridment with 1ry closure if it was Within 10–12 hours of injury on the
scalp and face

244. Spina bifida /what is the best diagnostic method?


A) MRI

• Correct Answer: A
UpToDate: The optimal study for characterization of intraspinal and peri-spinal
anomalies associated with Closed Spinal Dysraphism is MRI of the entire spine.

245. A 65-year-old male comes to the clinic with a mild intermittent urinary
flow reduction Rectal examination, urinalysis and prostate specific antigen
studies are normal (see report). Ultrasound prostate: Enlarged median lobe.
Which of the following is the best way to investigate?
A. Annual renal function monitoring
B. Periodic PSA measurement
C. Beta-blocker therapy
D. Cystoscopy

• Correct Answer: B
UpToDate: a PSA should be measured prior to initiation of treatment of BBPH with
5ARI, as such treatment may lower PSA levels (typically by 0.5 ng/dL) and influence
future prostate cancer screening. (Median Lobe enlargement = BBPH)

246. Patient known hypertensive on lisinopril and amlodipine, recently


diagnosed as TB
What will you do?
A) discountinue lisinopril
B) discontinue amlodipine
C( increase amlodipine

115
Correct Answer: C:

247. in order to eliminate dengue fever from Saudi Arabia with which you
should start
A- central
B- East
C- West

• Correct Answer: C
PubMed: Dengue fever (DF) is the most serious mosquito-borne viral
disease worldwide. DF is an acute febrile illness caused by Aedes aegypti and
Aedes albopictus, which are endemic in certain cities of Saudi Arabia, such
as Jeddah and Makkah (Mecca).

248. 65 y male heavy smoker medicaly and surgycally free


What screen for him ?
A) DM
B) HTN
C)AAA

• Correct Answer: C
AMBOSS: Men of advanced age are at increased risk for their formation; smoking
and hypertension are also major risk factors. AAAs are frequently asymptomatic
and therefore detected incidentally

249. pt came from southern east Asia with history of 2 weeks SOB cough
O/E Febrile + Rt upper lobe crepitation Xray ..upper Rt lobe opacity and
cavitation.. Best action:
A) ceftriaxone
B) sputum for AAFB
C) INH, rifampicin ,ethambutol ,Pyrazinamide

• Correct Answer: B

116
UpToDate: Active pulmonary TB often cannot be distinguished from
inactive disease on the basis of radiography alone. The detection of acid-
fast bacilli (AFB) on microscopic examination of stained sputum smears is
the most rapid and inexpensive TB diagnostic tool

250. 3 y old child diagnosed with malaria, what is type of malaria?


A) P falciparum,
B) P vivax,
C) P ovale
D) P malariae

• Correct Answer: A or B
both are common, but P falciparum are more dangerous and sever
I didn’t find any source that specify which is the commonest

251. 38 y male complain of constipation for 2 days painful perianal with


positive crack on 6 & 12 clock
A) Anal fissure
B) Perianal abscess

• Correct Answer: A
UpToDate: The majority of anal fissures are primary and are caused by local
trauma, such as constipation, diarrhea, vaginal delivery, or anal sex. The hallmark
symptom of anal fissures is pain. The pathognomonic feature of an acute fissure
(<8 weeks) is a superficial tear,

252. COPD patient came to clinic with bilateral lower limb edema and
pulmonary
hypertension. O2 sat 86%, PO2 8.6, PCO2 7.5, pH above normal range
Which of the
following the appropriate management to give for the patient now?
A) Start oral furosemide

117
B) Start oxygen therapy
C) Prednisolone therapy

• Correct Answer: B
UpToDate: Provide supplemental oxygen to target a pulse oxygen saturation of
92%
you should stabilize the patient first

253. Pt with K/C CKD , ... at the end asking what is Most common cause of
death in CKD?
A- CVD
B- hyperkalemia
C- Uremia
D- coagulation disorders

• Correct Answer: A
UpToDate: Patients with CKD, particularly those with kidney failure, are at
increased risk of mortality, particularly from CVD.

254. young male came to well baby clinic, upon examination his right testis
was palpable in the inguinal canal and small in size and easily moved to
scrotum, the left is normal, what is the cause?
A- ectopic testis
B- undescended testis
C- testicular torsion
D- Retractile testis

• Correct Answer: D
UpToDate: A nontraumatic, non-tender inguinal mass in the presence of an
"empty" hemi-scrotum is suggestive of a retractile testis if it can be brought back
into the scrotum.

118
255. 7 years child , systolic murmur heard in lower sternal area with no
thrill, (more noticeable when supine)
A) ASD
B) VSD
C) AORTIC STENOSIS
D) STILL MURMUR

• Correct Answer: D
UpToDate: Innocent murmurs Features
•Grade ≤ 2 intensity – Flow murmurs and innocent Still murmur) are usually grade
1 or 2 in intensity
•Softer intensity when the patient is sitting compared with when the patient is
supine
•Short systolic duration (not holosystolic, not diastolic)
•Minimal radiation – The murmur is located in a limited region of the precordium
•Musical or vibratory quality

256. Pt's friend ask you about diagnosis of Pt's disease. You refused. What is
ethical concept?
A) privacy
B) confidentiality
C) dignity
D) another option

• Correct Answer: B
UpToDate: Confidentiality refers to the "privileged and private nature of
information provided during the health care transaction"

257. pt of mesenteric ischemia ..best investigation?


A) CT
B) Exploratory Laparotomy
C) US

119
D) Laparoscopy

• Correct Answer: A
AMBOSS: + UpToDate: : CTA is the preferred definitive diagnostic test

258. Male pt with features of SLE with active arthritis. Ask about best
management
A) methotrexate
B) HCQ
C) Methotrexate, HCQ & steroid
D) Another option

• Correct Answer: C
AMBOSS: consider as moderate disease because of joint involvement
hydroxychloroquine in addition to oral glucocorticoids with or without
immunosuppressive agents (methotrexate or azathioprine) which is the best
management not initial.

259. Patient has SLE with neurological manifestation patient already on


corticosteroids, what to add now?
A) cyclophosphamide
B) hydroxychloroquine
C) phenytoin
D) diazepam

• Correct Answer: A
Patients with severe or life-threatening manifestations secondary to major
organ involvement (eg, renal and central nervous system) generally require
an initial period of intensive immunosuppressive therapy (induction
therapy, ex: mycophenolate, azathioprine, cyclophosphamide, or rituximab)
to control the disease and halt tissue injury.

120
260. Pt has rest tremor but when he move to pick something the tremor
gone, but can’t do finger nose test with his right hand where is the lesion?
A) Right cerebellum
B) Left cerebellum
C) Right hemisphere

• Correct Answer: A
AMBOSS: intention tremor and positive finding with finger nose test are some
features of cerebellum lesion.
The cerebellum receives input from, and controls output to, the ipsilateral side of
the body, and damage to the cerebellum therefore results in deficits to the
ipsilateral side of the body.

261. Female patient presents With achalasia. what is the most appropriate
management?
A) Non surgical Pnemoatic dialation
B) And botilinium toxin imj
C) Surgical treatment
D) Hellar myotomy
E) CCB and Nitrate

• Correct Answer: D
UpToDate: + AMBOSS: if the question was about best effective method is hellar
myotomy.

262. Which is most indicative of malignant nodule rather than tuberculosis


nodule:
A) Hypercalcemia
B) Nodule size >5cm

• Correct Answer: B
UpToDate:

121
Clinical features associated with an increased probability of malignancy
include advanced patient age and underlying risk factors (smoking, chronic
lung disease).
Features of an incidental pulmonary nodule to be assessed on CT are:
1-Size: consistently among studies, size is an independent predictor for
malignancy.
2-Attenuation.
3-Growth.
4-Calcification pattern.
5-Border and lobar location.
6-Enhancement.

263. Child with foreign body aspiration?


A) Rigid endoscopy

• Correct Answer: A
UpToDate: rigid bronchoscopy is the procedure of choice to identify and remove
the object

264. Patient with HTN not responding for three anti hypertensive drugs on
Ultrasound the one kidney is bigger what is diagnosis
A) Adrenal hyperplasia
B) Renal artery stenosis
C) Pheochromocytoma

122
• Correct Answer: B
AMBOSS: one of the findings in ultrasound for Renal Artery Stenosis is Ipsilateral
renal atrophy (decrease in kidney size) due to chronic ischemia. So the other
kidney will appear bigger,
Resistant hypertension (for 3 antihypertensive) also indicate this diagnosis.
If there’s supra-renal mass = Pheochromocytoma

265. Patient has complain about change of mood 3 days before


menstruation That affects her work and life What is the case :
A) PMDD
B) PMS

• Correct Answer: B
UpToDate: Criteria for PMS:
· One to four symptoms that may be physical, behavioral, or
affective/psychological in nature, or
· ≥5 symptoms that are physical or behavioral.

If, on the other hand, a woman has ≥5 symptoms and one of them is an
"affective symptom" (eg, mood swings, anger, irritability, sense of
hopelessness or tension, anxiety or feeling on edge), it is more accurate to
diagnose her with PMDD rather than PMS
AMBOSS:

123
266. RTA patient was transfused with 4 L of blood , which of the following
would be the complication?
A) Citrate toxicity
B) Hypocalcemia

• Correct Answer: A vs B (B is the obvious one and more critical)


UpToDate:
Hypocalcemia from citrate toxicity,
Blood is anticoagulated with sodium citrate and citric acid. As a result, massive
transfusion leads to infusion of a large amount of citrate. Infused citrate can lead
to a clinically significant fall in the plasma free calcium (ionized calcium)
concentration. This change can lead to paresthesias and/or cardiac arrhythmias in
some patients.

276. parents came with their 4 year old child who has not moved their right
arm for 2 days after falling on a stretched out arm. They claim on the first
day they saw no bruise or bleeding but thought its normal child not moving
the arm after falling. On the second day they got worried. On examination
the child looked well and healthy with no other injuries or signs of trauma.
On x-ray both ulnar and radial fractures. In the patient file you saw regular
visits with no concerns about the family?
A) Contact child services
B) Contact police
C) Treat the injury only (this option was worded differently but it suggested
no sign of child abuse so focus on injury)
D) i don’t remember

• Correct Answer: C
Explanation:
Parents were care of their child with regular check up visits
Fracture typically matching with full out stretching arm
So deal directly with the injury

124
268. 35-year-old pregnant women, classified as low risk pregnancy prior,
came today with vaginal bleeding and abdominal pain of which she was
diagnosed as placenta abruption. What is the next step?
A) continue same management.
B) classified her as high-risk patients and do U/S

• Correct Answer: B
UpToDate: Ultrasound examination is useful for identifying a retroplacental
hematoma and for excluding other disorders associated with vaginal bleeding and
abdominal pain.

269. Case after motor accident on examination of heart you hear muffled
heart sound (there is more but i forgot) Labs: 80/40 X-ray: normal
Diagnosis:
A) cardiac tamponade
B) hypovolemic shock

• Correct Answer: A
UpToDate: Beck's triad of cardiac tamponade:
Hypotension. Muffled heart sounds, Neck Vein distention (increased JVP)

270 Case of female child who lives in poor family with distended abdomen
and pitting edema there are lab results but I don’t remember them:
A) marasmus
B) Kwashiorkor

• Correct Answer: B
UpToDate:
Kwashiorkor (Deficiency Primarily in protein)
· Bilateral pitting edema, anasarca in severe cases
· Distended abdomen
· Hepatomegaly

125
· Rounded cheeks (moon face)
· Muscle atrophy

Marasmus (Deficiency in all nutrient)


· Profound muscle wasting (broomstick extremities)
· Loss of subcutaneous fat
· No edema

271. 27 years old women come to the ER complain of mild vaginal bleeding,
LMP was before 9 w, Ex revelead soft uterus felt above the symphysis pubic,
what is the most appropriate next step?
A) pelvic US
B) pelvic MRI
C) CT abdomen
D) CT chest abdomen pelvi

• Correct Answer: A
Hacker: Explanation: Pregnancy is suspected = bhcg and US first

272. Child had type 1 diabetes and consulting the family regarding celiac
screening , which of the following is true ?
A) Screen at diagnosis then every 5 years
B) Screen at diagnosis then every 2 years
C) Screen annually
D) Screen at diagnosis then annually for 5 years

• Correct Answer: B
UpToDate: If the screening test is negative (At time of T1D diagnosis), patients
should be rescreened for celiac disease within two years of diagnosis and then
again after five years.
More frequent screening and repeat screening after five years of diagnosis is
appropriate for children who develop symptoms suggestive of celiac disease

126
(gastrointestinal symptoms, poor growth, weight loss, or increased occurrence of
hypoglycemia) or for those who have a first-degree relative with celiac disease

273. Lung node meassure 8mm asymptomatic what to do?


A) Follow up
B) CT scan
C) biopsy
D) review previous x-ray

• Correct Answer: D
Imaging — Imaging assesses the size, attenuation, growth, and metabolic
activity of an incidental nodule. Every effort should be made to obtain prior
images that could have included the relevant area. Growing nodules are
suspicious for malignancy whereas long-term stability suggests a benign
etiology.
If question ask about diagnostic = CT Scan the preferred modality

274. case of PCOS asking about treatment.


A) Letrizole
B) tamoxifin
C) Progesterone
D) 4th one was wrong

• Correct Answer: A
AMBOSS:
Patients planning to conceive:
(Letrozole: first-line therapy for ovulation induction, Clomiphene: alternative to
letrozole)
(Exogenous gonadotropins: The low-dose regimen is the second-line treatment for
ovulation induction.)

Patients not planning to conceive:


(Combined oral contraceptives: first-line therapy)

127
275. indications for therapeutic thoracocentesis
A) Loculated
B) Ph > 7.2
C) High glucose

• Correct Answer: A
UpToDate: Therapeutic thoracentesis is commonly performed for symptom relief
(eg, dyspnea) or if the fluid has imaging characteristics of a complicated pleural
effusion (eg, loculations suggesting a parapneumonic pleural effusion). Therapeutic
thoracentesis is also indicated for patients with pleural conditions that risk pleural
thickening and restrictive functional impairment, such as effusions due to post-
primary or reactivation tuberculosis and hemothorax.

276. Polyhydramnios cause ?


A) Anencephaly
B) Post term pregnancy
C) Maternal ingested NSAIDs
D) Posterior urethral valve

• Correct Answer: A
AMBOSS: anencephaly (leads to impaired swallowing of amniotic fluid, leakage of
cerebrospinal fluid, and increased urination due to lack of fetal ADH

277. 24 years old male, medically and surgically free , presented with
manifestation of intestinal obstruction, suspected to be small bowl
obstruction
What is the most common cause
A) hernia
B) adhesion
C) meckles diverticulum
D) malignancy

128
• Correct Answer: A
UpToDate: Mechanical small bowel obstruction is caused by intraluminal or
extraluminal mechanical compression. Adhesion is the most common cause,
followed by hernias, malignancies, and various other infectious and inflammatory
disorders
The patient is surgically free so adhesion is excluded

278. 22 years female known case of severe depression ،suicidal attempts,


she is controlled on paroxetine Now, she is pregnant. What to do:
A) Stop paroxetine because of fetal malformation
B) Continue paroxetine and control her depression
C) Switch to others drug i think
D) Stop paroxiten b/c of prematurity

• Correct Answer: mostly C


UpToDate: We do not consider one SSRI to be safer or less safe than another to
use during pregnancy, with the possible exception of paroxetine. Several studies
have found that paroxetine was associated with a small increased risk for
congenital cardiovascular malformations; however, other studies have found no
such risk.

279. 25 years old primigravida 20 weeks GA history of mitral stenosis due to


rheumatic fever What physiological change makes her at high risk for heart
failure?
A) increase RBC mass
B) increase stroke volume
C) increase minute ventilation
D) increase Renal plasma volume

• Correct Answer: B
UpToDate: In Mitral Stenosis, the stenotic mitral valve restricts diastolic left
ventricular filling, resulting in an elevated transmitral gradient and left atrial
pressure that are further increased by the physiologic hypervolemia and increased

129
heart rate during pregnancy, thereby increasing the risk of pulmonary congestion
or pulmonary edema.

280.

A) Uterine Fibroid (leiomyoma)

UpToDate: Fibroids are seen on ultrasound usually as hypoechoic, well-


circumscribed round masses, frequently with shadowing; cellular fibroids may
appear to be more isoechoic, making differentiation from the normal myometrium
difficult, or hyperechoic

281. Pregnant at 3rd trimester with asymptomatic bacteriuria:


A) amoxicillin
B) nitrofurantoin

• Correct Answer: B
UpToDate: it’s only avoided in first trimester
It’s the first-line treatment for asymptomatic bacteriuria

130
AMBOSS: in general this treatment not recommended in (Healthy,
nonpregnant women, Older adults and individuals with diabetes, indwelling
catheters, or spinal cord injuries)

282. 36 year old male at ER C/O Right abdominal Pain , O/E :


fever, anorexia , weight loss ,
tenderness in RQ and Lower intercostal margines also patient is toxic
Temp. 37.9 ( I think but it was elevated ) wbc high, bilirubin high US :
cystic lesion without septates CT : homogenous
(not sure) and “THICK WALL with Peripheral enhancement
what’s most appropriate Mx :
A) Early laparoscopic cholecystectomy
B) Emergent stent chole
C) Cholecystectomy after 3 months

• Correct Answer: B
case of liver liver abscess
UpToDate: If CT scan is done, it should be with intravenous contrast, if possible.
The most typical finding is a well-defined, round lesion with central
hypoattenuation. Peripheral rim enhancement or surrounding edema are not
common findings but are specific for liver abscess.
Drainage of the abscess contents is a standard component of therapy.
Drainage techniques include computed tomography (CT)-guided or ultrasound-
guided percutaneous drainage (with needle aspiration only or with catheter
placement), open surgical drainage, laparoscopic drainage, or drainage by
endoscopic retrograde cholangiopancreatography (ERCP).

283. 5 years old man Hx of laproscopic appendectomy 10 years ago, present


to ER with sudden abdominal pain and vomiting , mild generalized
tender, vitally stable, abdomen x ray shows multiple air fluid level and
dilated loop in certian point, what is next step?
A) Diagnostic laparoscopy
B) laparotomy
C) CT abdomen

131
D) barium swallow

• Correct Answer: C
AMBOSS: CT abdomen and pelvis is gold standard for bowel obstruction

284. Patient presented with dysphagia to liquids more than solid


Which is the most appropriate initial investigation:
A) endoscopy
B) barium swallow
C) US
D) Biopsy

• Correct Answer: A
UpToDate:

132
285. Patient with rectal bleeding 5 ; 7 o clock sclerotherapy done What type
of hemorrhoid treated:
A- internal
B- external
C- prolapse
D- thrombosed

• Correct Answer: A
UpToDate: Sclerotherapy - Injectable sclerosant solutions can also be used to treat
symptomatic internal hemorrhoids.

133
286. Pt bring her 7 yrs old child worried about short stature
There is delayed bone growth
What is management:
A) reassure he will gain adult stature with puberty
B) do basic investigations cbc.TFT.bone age.

• Correct Answer: B
I’m not sure, but according to UpToDate if there’s a delay in bone growth we
should investigate more.
Unfortunately we have 2 options only.

287. CTG sinusoidal pattern


Management??
A) Cord prolapse > Cs

• Correct Answer: A
UpToDate: the pattern has been associated with fetal anemia and some maternal
narcotics. Delivery is generally indicated if resuscitative measures do not improve
the pattern.

288. Serum K 6.1 Management?


A) Ca gluconate

• Correct Answer: A
AMBOSS: If there are ECG changes: Stabilize the cardiac membrane first (IV
calcium gluconate)

289. And what ECG change for hyperkalemia ?


A) Peak T wave

• Correct Answer: A (AMBOSS)

134
290. Pt admited to ICU with inferior MI started ttt then brady cardia i think
What is the diagnosis:
A) first degree heart block
B) second degree heart block

• Correct Answer: B
UpToDate: Inferior MI - Conduction disturbance in inferior MI may occur upon
presentation or after hours or days. Sinus bradycardia, Second Degree Heart
degree Mobitz type I (Wenckebach), and complete heart block (CHB) are
commonly seen, since the SA node, AV node, and His bundle are primarily supplied
by the right coronary artery

291. Patient post cholecystectomy presented with abdominal pain .fever Rt


pleural effusion Collection at gallblader fossa ..what is the apropriate
management:
A) US aspiration
B) ERCP

• Correct Answer: B
UpToDate: + schwartz’s:
Mostly it’s biliary injury leading to fluid collection in the gallbladder fossa = biloma
Most injuries (biliary injury) are not recognized at the time of initial surgery. Early
presentation of bile duct injury after laparoscopic cholecystectomy is generally
nonspecific, with patient complaints of vague
abdominal pain, persistent nausea and vomiting, and low-grade fever
In type A injuries (which is the most common), the stent can be removed
endoscopically at two weeks if the patient is asymptomatic, the liver function tests
are normal, and there is no ongoing leak at the follow-up ERCP.

292. Patient with Rt upper quadrant pain...jaundice fever...O/E RUQ


tenderness, High TWBCs, Amylase normal, Mildly dilated CBD 1.3. What is
the apropriate diagnosis:
A) pancreatitis

135
B) cholangitis
C) appendicitis
D) acute cholecystitis

• Correct Answer: B
AMBOSS: Charcot cholangitis triad (25–70% of patients present with all
three features)
Abdominal pain (most commonly RUQ)
High fever
Jaundice

293. Patient with Rt upper quadrant pain...fever...jaundice..hx of stone O/E


generalised abdominal tenderness and sluggish bowel sounds, High TWBCs,
High amylase, US showed mildly dilated CBD 1.1 What is the apropriate
diagnosis:
A) pancreatitis
B) cholangitis
C) appendicitis

• Correct Answer: B
AMBOSS: Charcot cholangitis triad (25–70% of patients present with all three
features)
Abdominal pain (most commonly RUQ)
High fever
Jaundice

294. Appropriate management of papillary thyroid Cancer ?


A) total thyroidectomy

• Correct Answer:
Current + UpToDate:
For solitary papillary carcinomas less than 1 cm, thyroid lobectomy is adequate
treatment.

136
Total thyroidectomy is recommended for papillary (> 1.0 cm)

295. Scenario of patient with nephrotic syndrome then developed


generalized abdominal pain and tenderness Which of the following describe
complication occurred:
A) peritonitis
B) pancreatitis

• Correct Answer: A
AMBOSS:
Peritonitis: Generalized Abdominal pain/tenderness, Peritoneal signs (e.g., local or
diffuse rigidity, rebound tenderness, and/or guarding)
Pancreatitis: epigastric pain radiating to the back

296. RTA patient came with deformed swell leg...there is pain and
paresthesia and absent pulse Most appropriate management:
A) Fasciotomy

• Correct Answer: A
Acute compartment syndrome: predominantly trauma-induced; a surgical
emergency
Fasciotomy (tissue and fascia incisions): relieves the pressure, thus restoring
perfusion.

297. 14 years old female with infrequent vomiting during her period_came
this time with vomiting associated with smal amount of blood then
symptoms releived. no vomiting..soft non tender abdomen Most apropriate
next step:
A) Admit for observation
B) Discharge home and tell to come if symptoms recur
C) Prepare for urgent endoscopy

137
• Correct Answer: B
Mallory Weiss syndrome is suspected because of recurrent vomiting
UpToDate: Patients without risk factors for rebleeding, evidence of severe
upper gastrointestinal bleeding, or endoscopic stigmata of recent bleeding
can be discharged from the hospital on oral antisecretory therapy once the
effects of procedural sedation have worn off, provided that the patient is
reliable and can promptly get medical care should bleeding recur.
Hospitalization only if she’s high-risk for rebleeding.

298 Child point to red color..ride tricycle. Dress himself what is his age??
A) 4 years

• Correct Answer: A
Nelson + Illustrated Textbook of pediatrics:
Tricycle > 3yrs
Knowing colors + dressing > 4 yrs
So 4 yrs

299. Rheumatoid arthritis patient on NSAID presented with sever epigastric


pain and tenderness Most appropriate investigation:
A) Erect CXR
B) US abdomen

• Correct Answer: A
It’s important to rule out perforation (the patient is on NSAID and this
consider as a risk for gastritis and gastric ulcer)

300. Pic of Peripheral blood film show microcytic hypochromic cells with
scenario ...HB low/ RBCs low What suspect to be low:
A) MCV
B) Reticulocytes
C) Platelets

138
• Correct Answer: A
AMBOSS:
We suspect the patient has iron deficiency anemia so the mcv will be low

301. Patient heavy smoker presented with SOB and wheeze. Senario of
COPD
What immediate inhaler to give:
A) ipratropium bromide
B) Ventolin

• Correct Answer: B
UpToDate: We recommend that all patients with a COPD exacerbation receive
inhaled short-acting bronchodilator therapy.are the mainstay of therapy for an
acute exacerbation of COPD because of their rapid onset of action and efficacy in
producing bronchodilation,

302. Mother 2 months post-delivery. asymptomatic. culture screening of


urine more than100000 E.coli sensitive to ciprofloxacin nitrofurantoin anf
trimethoprim sulphamethoxazole Ask about treatment:
A) ciprofloxacin
B) nitrofurantoin
C) trimethoprim sulph...
D) no need

• Correct Answer: D
UpToDate: :
• This is case of Asymptomatic Bacteriuria
No need for treatment (treat only if: pregnant, recent kidney transplant,
expected to go through urinary procedure)

139
303. Young patient complain of suprapubic pain and dysuria
No fever
Urine analysis given=
Urine colour yellow
Turbid
Leukocytes 10
Other normal
What is the next step:
A) empirical antibiotics
B) do culture and wait for result
C) no need for treatment

• Correct Answer: A
AMBOSS: Symptomatic UTI
Initial treatment is with an empiric regimen, which is maintained for
uncomplicated cystitis

302. female 15 days post c/s presented with suprapubic pain ,fever and
vaginal discharge O/E wound clean Diagnosis:
A) endometritis ....

• Correct Answer: A
UpToDate: Cesarean birth is the dominant risk factor for development of
postpartum endometritis.
Criteria of postpartum endometritis:
-Fever (≥100.4°F [38°C])
-Pain or tenderness (uterine or abdominal)with no other recognized cause
-Purulent drainage from the uterus

303. Younge female presented with suprapubic pain and heavy vaginal
discharge O/E tender fornixes what diagnosis:
A) cervicitis
B) vaginitis

140
C) endometritis
D) Salpingitis

• Correct Answer: D
UpToDate: Patients with acute symptomatic PID generally complain of recent
onset of lower abdominal pain in association with new vaginal discharge and/or
intermenstrual bleeding. Constitutional symptoms may occur and include fever
and chills. Perihepatitis can also occur and present with marked tenderness in the
right upper quadrant. Pelvic organ tenderness on palpation is the defining exam
finding.

311. Scenario post deliver pph then ask about management of PPH:
A) misoprostol
B) propofol
C) dexamethasone
No oxytocin in answers

• Correct Answer: A
UpToDate: here is the approach of medication (I think it’s deficit question)
Medications used in PPH Management
1) Oxytocin 10-40 units in 500/1000ml NS/RL or 10 units IM (1st line)
2) Methylergonovine 0.2 mg IM (2nd line, Contraindicated in preeclampsia
and HTN)
3) Carboprost (hemabate) PG F2a 0.25mg IM(3rd line, Contraindicated in
Asthma)
4) Misoprostol 600-1000 micrograms PO, or rectal (4th line)
We can use tranexamic acid first (it was found that reduced death due to
bleeding in patients with PPH by 20 to 30 percent)

312. Asthmatic patient on ICS and use SABA inhaler at least once daily What
to add to his medications:
A) leukotriene antagonist
B) salmeterol

141
• Correct Answer: B
AMBOSS:

According to Stepwise pharmacological treatment of chronic asthma

313. Child with sore throat..toxic with drooling..sitting upright with mouth
open
What is the causative organism:
A) hemophilus influenza type B
B) parainfluenza
C) Rhinovirus
D) Adenovirus

• Correct Answer: A
Explanation: this is a case of Epiglottitis (tripod position – sitting upright, drooling
saliva)
AMBOSS: + UpToDate: the most common causative pathogen is hemophilus
influenza type B

142
314. Child with staccato cough_inspiration between cough and post tusive
vomitiyn_ there is history of conjectivitis and eosinophilia in investigations
what is the diagnosis:
A) pertussis
B) chlamydia pneumonia
C) mycoplasma pneumonia

• Correct Answer: B
AMBOSS: staccato cough is characteristic for chlamydia pneumonia, Typically
afebrile and Accompanied by neonatal conjunctivitis in up to 50% of all cases.

315. Pt on oral steroids presented with white layer in his toungue and
buccal mucosa what is the treatment:
A) nystatin
B) ketoconazole
C) Amphotericine B

• Correct Answer: A
AMBOSS: one of adverse effects of oral steroid is oral candidiasis (thrush), it’s
White plaque in the oral cavity that can be scraped off.
Treatment: First line: topical nystatin or oral fluconazole

317. Pt known case of chronic hepatitis B and asymptomatic for follow up


US showed liver with mild coarse texure and nodule 3×3 what most
apropriate investigation:
A) alfa feto protien
B) biopsy
C) US abdomin
D) triphasic abdominal CT

• Correct Answer: D

143
UpToDate: For patients at high risk for developing HCC, the diagnosis can be made
with dynamic contrast-enhanced computed tomography (CT) or magnetic
resonance imaging (MRI) tailored for liver lesion evaluation

318. 4 yrs child contact with TB patient..PPD skin test 10mm:


A) Positive
B) negative
C) repeat test
D) no need for test

• Correct Answer: A
AMBOSS:

319. Pt 64 Male , HT , Diabetes, HF , history of stroke the CHADS2 is ?


A) 3
B) 4
C) 5

• Correct Answer: C

144
320. 62 y with bph , BMI 41 , what is the risk of BPH in this case:
A) age
B) obesity

• Correct Answer: A
AMBOSS: Prevalence of BPH increases with age (present in ∼ 50% of men > 50
years and more than 80% of men > 80 years)

321. Pt did upper endoscopy and there is squamous carcinoma Which of


the following is a risk factor of esophageal cancer?
A) GERD
B) Barret’s esophagus
C) Esophageal stricture
D) smoking

• Correct Answer: D
AMBOSS:
The primary risk factors for squamous cell esophageal cancer are alcohol
consumption, smoking, and dietary factors
The most important risk factors for esophageal adenocarcinoma are
gastroesophageal reflux and associated Barrett esophagus.

145
322. Long case: Most significant risk factor for MI?
A) smoking
B) age
C) hypertension

• Correct Answer: C
Explanation: The most common risk factor among patients with initial MI was
hypertension (52.3%), followed by smoking (31.3%), dyslipidemia (28.0%), family
history of CHD (28.0%), and the least common traditional risk factor, diabetes
(22.4%)

323. Mother brings her infant to family medicine clinic for regular check,
the doctor advice her to start iron supplement after 1 month from this visit,
from this scenario her child in which age now (in months)?
A) 1
B) 2
C) 3
D) 6

• Correct Answer: C
UpToDate: Breastfed infants require some form of iron supplementation (iron-
fortified infant cereal, puréed meats, iron-rich vegetables, liquid iron supplements)
beginning at four months of age to meet their iron requirement (1 mg/kg per day).
Infants who receive iron-fortified formula do not require additional iron
supplementation.

324. Baby Preterm diliver (didn't specify CS or SVD). 4 hours later


complaining from SOB
Tachypnea, Tachycardia with grunting

A) Hyaline membrane disease


B) Pulmonary "something"
C) Meconium Aspiration syndrome

146
D) Transient Tachypnea of the New born

• Correct Answer: A
Explanation:
Hyaline membrane disease is primarily seen in preterm neonate and
present soon after birth
UpToDate:
Infants with RDS are nearly always preterm. Respiratory distress (ie,
tachypnea and labored breathing) and cyanosis occur at or soon after birth.
Typical signs include grunting (which prevents end-expiratory alveolar
collapse), nasal flaring (which reduces nasal resistance and reflects
increased use of accessory muscles of respiration), and intercostal and
subcostal retractions (due to decreased lung compliance and the highly
compliant chest wall)

VS

TTN is most frequently seen in late preterm infants born at a gestational


age between 34 and 37 weeks, many of whom are delivered by elective
caesarean section [8]. Term and postterm babies are also at risk for TTN.

325. Pregnant with recurrent UTI what to do?


A. X ray
B. US
C. ureteroscopy
D. Cystoscopy and RFTs

• Correct Answer: B
UpToDate:
Almost related answer:
Imaging is not routinely used to diagnose pyelonephritis. However, in patients with
pyelonephritis who are severely ill or who also have symptoms of renal colic or
history of renal stones, diabetes, history of prior urologic surgery,
immunosuppression, repeated episodes of pyelonephritis, or urosepsis, imaging of
the kidneys can be helpful to evaluate for complications. In pregnant women, renal

147
ultrasound is the preferred imaging modality in order to avoid contrast or radiation
exposure.

If symptoms and fever persist beyond the first 24 to 48 hours of treatment, a


repeat urine culture and renal ultrasound should be performed to rule out
persistent infection and urinary tract pathology.

326. What is the most common cause of bleeding in postmenopausal


women ?
A) endometrial atrophy
B) endometrial cancer

• Correct Answer:A
UpToDate: The differential diagnosis of bleeding in postmenopausal patients is less
broad than that for abnormal bleeding in premenopausal patients since the various
causes of anovulation are not relevant. In a prospective study including 454
postmenopausal patients with uterine bleeding, the frequency of endometrial
pathology was as follows [6]:
●Polyp (37.7 percent)
●Hypotrophy/atrophy (30.8 percent)
●Proliferative/secretory (14.5 percent)
●Carcinoma (6.6 percent)
●Fibroid (6.2 percent)
●Hyperplasia without atypia (2 percent)
●Hyperplasia with atypia (0.2 percent)

327. 2 Female pt has Bloody nipple discharge what initial breast


investigation?
A) Mammo

• Correct Answer: Mammo Vs US?

148
UpToDate: Mammography and ultrasound — For patients who present with
pathologic nipple discharge, we usually start with diagnostic mammography
and/or focused breast ultrasonography.

AMBOSS: <30 years old -> US


>30 years old -> Mammo

328. Another recall: 32 year old female came with bloody nipple discharge
which of the following initial diagnostic tole?
A. US
B. mammogram

• Correct Answer: B

329. Pregnant case of epilepsy for 2 years and poor control despite
medication, what will you do?
A) Review the medication
B)
C)
D)

330. Most causative organism in case of valvular replaced patient,


vegetation seen?
A) strep viridians, strep mitis

Explanation: This mostly depends on the duration since implantation of the


prosthetic valve
UpToDate:
During the initial two months of implantation, the most frequently
encountered pathogens were S. aureus and coagulase-negative
staphylococci (CoNS); next in frequency were gram-negative bacilli and
Candida species. This spectrum of organisms reflects the typical nosocomial
origin of these infections.

149
Between 2 and 12 months after implantation, the most frequently
encountered pathogens were coagulase-negative staphylococci, S. aureus,
and streptococci, followed by enterococci. In general, cases occurring 2 to
12 months after surgery are a blend of delayed-onset nosocomial and
community-acquired infections.

AMBOSS:
• Early-onset: Coagulase-negative staphylococci, S. aureus, or gram-
negative bacilli (most common) [15]
• Late-onset: Coagulase-negative staphylococci, S. aureus, Viridans group
streptococci (most common) [15]

331. What of the following will cause oligohydramnios?


A) Placental insufficiency
B) DM
C) Chriongioma
D) Dudenal atresia

• Correct Answer: A
Explanation: Polyhydramnios is caused by DM & Duodenal atresia

332. Old male pt did total thyroidectomy then he developed swelling in the
neck with inspiratory stridor and sob How u will manage
A) thoracotomy
B) surgical exploration
C) drainage
D) Surgical evacuation

• Correct Answer: D
UpToDate: If a hematoma develops, it should be evacuated in the operating room.
In a retrospective study of 207 patients who developed a hematoma after
thyroidectomy, 79 percent required operating room evacuation within 24 hours of
the original surgery. Patients diagnosed with a hematoma should be kept awake

150
and promptly transported to the operating room for evacuation. In the operating
room, in most cases the patient's neck should be cleansed with antiseptics, the
incision opened, and blood from the hematoma evacuated all prior to intubation,
as a large cervical hematoma might make intubation more difficult by compressing
the larynx.

333. Child has fatigue and splenomegaly, Hb low, RBC low , MCV low , Retic
normal , Iron normal , What to replace in this case ?
A) Iron
B) B12
C) Folate
D) Erythrocytes

• Correct Answer: D
Explanation: Low Hb and RBCs with normal (or increased) reticulocytes indicate
hemolysis + iron is normal

334. Pt. diagnosed with lung cancer what will do before operation?
A) Chest x Ray
B) Ecg
C) Cbc
D) Lung function test
E) Echo

• Correct Answer: All is indicated but Lung function test is required to


determine prognosis after operation so most likely is D

UpToDate: We agree with guidelines that the forced expiratory volume in one
second (FEV1; ie, spirometry) and the diffusing capacity for carbon monoxide
(DLCO) should be measured in all patients in whom resectional surgery is being
considered

AMBOSS: Preoperative assessment: Preoperative pulmonary function testing is


required prior to lung resection surgery

151
◦ A preoperative FEV1 < 1.5 L and DLCO < 60% predicts a poor outcome
after lobectomy.
◦ A preoperative FEV1 < 2 L and DLCO < 80% predicts a poor outcome after
pneumonectomy.

335. 55 years old male, no history of dm or any other disease


Bp: 159/75 What will give him?
A) lisinopril
B) amlodipine
C) hydrochlorothiazide
D) lisinopril + amlodipine

• Correct Answer: D
UpToDate:
Single-agent therapy will not adequately control blood pressure in most patients
whose baseline systolic blood pressure is 15 mmHg or more above their goal.
Combination therapy with drugs from different classes has a substantially greater
blood pressure-lowering effect than doubling the dose of a single agent, often with
a reduction in side effects seen with a higher dose of monotherapy [60]. When
more than one agent is needed to control the blood pressure, we recommend
therapy with a long-acting ACE inhibitor or ARB in concert with a long-acting
dihydropyridine calcium channel blocker.

AMBOSS:
Patient's initial blood pressure:

SBP 130–139 mm Hg or DBP 80–89 mm Hg (stage 1 hypertension): Consider initial


monotherapy.

SBP ≥ 140 mm Hg or DBP ≥ 90 mm Hg AND an average blood pressure > 20/10 mm


Hg above target -> Initiate combination therapy.
Commonly used combinations are an ACEI or ARB PLUS either a dihydropyridine
CCB OR a thiazide-type diuretic.

336. Tracheomalacia highest diagnostic value?

152
A) Laryngoscopy.. Bronchoscopy Flexible bronchoscopy
UpToDate: Definitive diagnosis usually is made by bronchoscopy during
spontaneous breathing [74,85]. With this technique, the trachea is observed to
collapse during expiration. Noninvasive diagnosis of this disorder may be improved
by newer imaging techniques, including multidetector computed tomography (CT,)
in which end-expiratory and end-inspiratory images are obtained

337. Another recall:


*Sx of tracheomalacia .. Dx??
A-fluoroscopy
B-rigid bronchoscopy
C-flexible bronchoscopy

• Correct Answer: C
UpToDate:
Dynamic airway endoscopy is the diagnostic tool of choice. Changes in airway
caliber during fluoroscopy and contrast bronchography occasionally can establish a
diagnosis but are less commonly used than endoscopy

Diagnostic tests — Diagnostic modalities that are used to evaluate suspected TM


include dynamic flexible bronchoscopy (DFB), dynamic airway computed
tomography (DACT), and pulmonary function testing (PFT).

338. newborn, abdominal distention and palpable bilateral kidney.


Distended proximal urethra, ask about Dx?
A) bilateral hydronephrosis
B) cystic diseases
C) due to malignant or benign infiltrative disease

Another recall:
Newborn, abdominal distention and palpable bilateral kidney.
Distended proximal urethra, ask about Dx?

153
• Correct Answer:A
Explanation: Bilateral hydronephrosis due to Posterior urethral valve is the
most common cause of urinary tract obstruction in newborn males,
associated also with abdominal distension.
AMBOSS:
Most common cause of urinary tract obstruction in newborn males
Respiratory distress secondary to pulmonary hypoplasia in cases with severe
obstruction (see “Potter sequence”)
Abdominal distention due to bladder distention
Late manifestations:
Difficulty voiding, poor urinary stream
UTIs → urosepsis
Diurnal enuresis
Failure to thrive

339. Last mensural cycle was (?/?/2019) , what’s the estimated date of
confinement by Naegele rule?

Naegele rule: used to calculate the expected date of delivery (due date)
First day of the last menstrual period + 7 days + 1 year - 3 months

340. Patient takes retinoic acid asking when she can get pregnant
A) Tell her to stop retinoic acid now and try to get pregnant
B) She can get pregnant after she finish the course
C) get pregnant during the course no problem
D) She can get pregnant after 3 months of stopping retinoic acid

• Correct Answer: D
UpToDate: Topical retinoid therapy is not recommended in pregnancy.
AMBOSS:
Precautions (in all females of childbearing potential)
1 month before initiating therapy:
A serum/urine pregnancy test

154
Two methods of contraception (oral contraceptive therapy + barrier
contraception / IUD)
During therapy: monthly pregnancy test and continuous use of two
methods of contraception
After completing therapy:
Continue two methods of contraception for 1 month
A pregnancy test at the end of 1 month
“To not get pregnant for at least 1 month before you start taking the
medicine, while you’re taking the medicine or for at least 1 month after you
stop taking the medicine”
https://www.marchofdimes.org/pregnancy/isotretinoin-and-other-
retinoids-during-pregnancy.aspx

341. Female with bloating and GI symptoms, started gluten free diet by
herself and now she is better and visiting the GP. What to do?
A) Dietician referral
B) GI referral
C) No need to continue on gluten free diet
D) Stop gluten free diet and then do celiac serology

• Correct Answer: D
UpToDate: While there are limited data with regard to the decrease in antibody
titers in individuals on a gluten-free diet, a weakly positive test may become
negative within weeks of strict adherence to a gluten-free diet [50]. After 6 to 12
months on a gluten-free diet, approximately 80 percent of individuals with celiac
disease will test negative by serology. By five years, more than 90 percent of
patients on a gluten-free diet will have negative serologies [51,52]. An approach to
diagnosis of celiac disease in patients on a gluten-free diet
●Baseline evaluation and gluten challenge – Baseline antibody testing should be
performed. Patients with positive serology should undergo a small bowel biopsy
AMBOSS:
Diagnosis is based on serology (initial testing) and EGD with duodenal biopsy
(confirmation).
A gluten-free diet can cause negative test results.

342. Pediatric pt have all UTI Sx and ask what’s the most diagnostic value?

155
A) Two mixed organisms from clean catch
B) Single organism from any colony from mid-stream urine sample
C) Single organism (3x10^5) from suprapubic

• Correct Answer:C
UpToDate:
The technique for urine collection is primarily determined by whether the patient
is toilet trained or not. In infants and children who are not toilet trained,
transurethral bladder catheterization is the most commonly performed technique.
However, suprapubic aspiration is least likely to result in a contaminated urine
culture.
We recommend that infants and children with a suspected urinary tract infection
(UTI), who are not toilet trained and who are ill enough to merit antimicrobial
therapy, have urine cultures obtained by TUBC or SPA rather than by clean catch or
clean urine bag specimen [6,7]. We usually perform TUBC in our patients. Although
specimen contamination is more likely by TUBC than by SPA [1,3,6,8], urine is also
more likely to be obtained by TUBC [9,10]

343. Pregnant woman found to be non-immune to rubella, when to give


vaccine?
A) 3rd trimester
B) After delivery

• Correct Answer: B
Explanation: Rubella vaccine is live attenuated which makes it
contraindicative during pregnancy

344. child came with nasal congestion on examination and there is pale
nasal polyp and dark periorbital swelling
A- allergic rhinitis
B- rhinitis medicamentosa
C- Acute bronchitis
D- foreign body

156
• Correct Answer:A
UpToDate: Allergic rhinitis presents with paroxysms of sneezing, rhinorrhea, nasal
obstruction, and nasal itching. Postnasal drip, cough, irritability, and fatigue are
other common symptoms [1-3]. Some patients experience itching of the palate and
inner ear. Those with concomitant allergic conjunctivitis report bilateral itching,
tearing, and/or burning of the eyes
AMBOSS:
Recurrent episodes of sneezing, nasal congestion, rhinorrhea, and postnasal drip
Itchy nose and throat
Pale, boggy nasal mucosa with hypertrophic turbinates
Nasal polyps are seen in 25–30% of patients with chronic allergic rhinitis. [8]
Cobblestone appearance of the posterior pharyngeal wall
Allergic shiners: hyperpigmentation and edema of the lower eyelid as a result of
venous congestion

345. female 72 year C/o knee pain,morning stiffness, wake her at night
what is the Diagnosis?
A) Osteoarthritis
B) Osteoporosis
C) Rheumatoid

• Correct Answer: C
UpToDate: Morning stiffness is a common feature of those with active RA; it can
be defined as "slowness or difficulty moving the joints when getting out of bed or
after staying in one position too long, which involves both sides of the body and
gets better with movement".

346. 6yrs with inability to bear weight on exam there was bilateral knee
pain. Labs showed positive ANA. Juvenile onset arthritis is suspected what
type could he have?NB:Oligoarthritis JIA

AMBOSS:
Arthritis involving ≤ 4 joints within 6 months of disease onset + Large weight
bearing joints

157
347. here was a case accidentally finding in health campaign a pediatric
patient with 5cm thyroid nodule, TSH is normal, no compressive symptoms
what’s the appropriate next step:
A- radioactive iodine
B- FNA
C- nothing

• Correct Answer:B
UpToDate:
FNA is the most useful test to differentiate benign thyroid nodules from cancer and
is preferred over core biopsy, which is more invasive and has a higher risk of
complications. FNA has high diagnostic accuracy in children [46-49]. However, the
size criteria for performing FNA in adults (≥1 to 1.5 cm) may not be appropriate in
a growing child whose thyroid gland may normally be one-half the size of that of
an adult. Therefore, the American Thyroid Association Task Force recommends
that "ultrasound characteristics and clinical context should be used rather than size
alone to identify nodules that warrant FNA" [50].
Accordingly, we perform FNA biopsy in children on nodules with the following
characteristics:
●Nodules ≥1 cm (palpable or nonpalpable) that are solid or mixed
cystic/solid.
●Nodules <1 cm with ultrasound characteristics that are highly suspicious
for cancer, such as calcifications or abnormal cervical lymph nodes

348. Dysmenorrhea, how to best diagnose endometriosis:


A. laparoscopically
B. USC.
C. MRI

• Correct Answer:A
AMBOSS:
Laparoscopy (confirmatory test) may show endometriotic implants and adhesions

349. Patient came with vaginal discharge she have done CS with episiotomy
10 days ago, the obstetrician diagnosed her with UTI and described Abx ,

158
but she did not improve then she went to another obstetrician and he
found infected vaginal swab , What is the medical error done by the first
obstetrician?
A. let the midwife assist him and depend on her
B. doctor failed to follow the surgical safety protocol in the OR
C. no communication between the second and first obstetrician
D. genuine differente assessment or diagnosis of patient case

• Correct Answer:B

350. Woman with hx of miscarriage, now in 7 weeks with vaginal spotting,


no hx of passing tissue , on examination os is closed and no bleeding
A. Threatened abortion
B. Normal delivery
C. Incomplete abortion
D. Inevitable abortion

• Correct Answer: A
Explanation:
D. present with open cervix
C. present with some tissue in the cervix + open cervix
B. her Hx of miscarriage and early gestational age make it unlikely

AMBOSS:
A process of miscarriage that has started but not yet progressed to a state
from which recovery is impossible (potentially reversible) before 20 weeks'
gestation
Vaginal bleeding
Fetal activity
Closed cervical os

351. A baby with noisy breathing and wheezing that improves when prone
and increases when supine. What is the diagnosis?
A- Laryngomalacia

159
B- Tracheomalacia

• Correct Answer: A
AMBOSS:
Clinical features:
Usually happy and thriving infants
Inspiratory stridor: worsens in supine position, during crying, upper respiratory
tract infections, agitation, and feeding
Reflux may be present Failure to thrive and sleep-disordered breathing in severe
cases

352. Pediatric patient diagnosed as croup in the ER, he was given the proper
management, but after 1 hour he did not improve. What is the appropriate
management?
A-exposure to cool humidified air
B-prednisilone (with dose)
C-dexamethasone (with dose)
D-racemic epinephrine

• Correct Answer: D
Explanation: he was given “proper management” in the ER so we can assume they
used dexamethasone initially but without improvement, hence the most logical
next step is using racemic epinephrine as the second option

UpToDate:
Dexamethasone – We recommend glucocorticoid therapy for all children with
moderate to severe croup. Dexamethasone (0.6 mg/kg, maximum of 16 mg) is
generally the preferred glucocorticoid in this setting. Dexamethasone should be
administered by the least invasive route possible
Nebulized epinephrine – We recommend nebulized epinephrine in all patients with
moderate to severe croup

160
353. 9 y/o pt his mom complaining about he is wetting his bed since one
month. He used to be dry since 7years The urine is foul smelling His lab
showing high wbs and turbid in colour and showed nitrate Diagnosis ?
A- UTI
B- normal for his age
C- nocturnal enuresis

• Correct Answer:A
Explanation: foul smelling, high wbc, turbid in color are indicative of UTI. He is 9
years old so bed wetting is not normal for any child past the age of 5

354. pregnant patient on 20 week known case of SLE controlled on


Plaquenil what to do?
A) Refer to rheumatologist to stop medication
B) Stop medication
C) Continue medication

• Correct Answer:C
Explanation: Hydroxychloroquine is Plaquenil and its safe in pregnancy

355. They suspect hypovolemic AKI what you’ll see ?


A-BUN/Cr >20
B-FENA >2%

• Correct Answer: A
AMBOSS:

161
356. RT leg pale & absence of peripheral pulse + irregular irregular radial
pulse, What is the Source of thrombus?
A/ RT atrium
B/ LT ventricle
C/ Thoracic Aorta
D/ Abdominal Aorta

• Correct Answer:A

357. 35y.o man presented with fever, rigors & weight loss for 10 days. He
underwent Mitral valve replacement by prosthetic valve 1 month ago, Echo
showed a small vegetation. Vitals: Blood pressure 90/70 mmHg, HR
100/min, Temp 38.7°C. Caustive organisim is ?
A. Coxiella burnetli
B. Staphylococcus aureus
C. Streptococcus viridans
D. Staphylococcus epidermidis

• Correct Answer:B
UpToDate:
During the initial two months of implantation, the most frequently encountered
pathogens were S. aureus and coagulase-negative staphylococci (CoNS); next in
frequency were gram-negative bacilli and Candida species. This spectrum of
organisms reflects the typical nosocomial origin of these infections.

Between 2 and 12 months after implantation, the most frequently


encountered pathogens were coagulase-negative staphylococci, S. aureus,
and streptococci, followed by enterococci. In general, cases occurring 2 to
12 months after surgery are a blend of delayed-onset nosocomial and
community-acquired infections

162
358. Pt post some cholangio intervention few hours later he devolps chills
and rigor only no fever mentioned
TWBs was 9 normal till 10
RR 18
HR 90
Bp 115/88
Temp :38 degree exactly
What is the most likely diagnosis :
A) sepsis
B) SIRS
C) bactermia
D) septic shock

• Correct Answer: C
UpToDate:

Note: this patient does not fulfill the criteria that's why the answer here is C

359. 15 month boy presented with refusal of feeding vomiting febrile was
suspected to have meningitis so CSF was done showed normal glucose high
protein high cells, Mainly neutrophils, Culture showed gram, positive
diplococci, Which of the following is the most appropriate management :
A) ampicillin
B) ampicillin and gentamycin
C) Vancomycin
D) ceftriaxone and Vancomycin

• Correct Answer:D

163
AMBOSS:

360. Pediatric with seizure and other signs, labs showed metabolic acidosis,
urine analysis: aminoaciduria and glycosuria, and cataract what’s the
diagnosis?
A. Galactosemia
B. Homocystinuria
C. Methylmalonic Academia
D. Maple syrup urine syndrome

• Correct Answer: A
AMBOSS:

361. What is the structure having abnormality in cardiac tamponad :


A) pericardium
B) mayocardium
C) coronary artery

164
D) endocardium

• Correct Answer: A
AMBOSS:
Cardiac tamponade: pericardial fluid collection (e.g., bloody or serous) → ↑
pressure in the pericardial space → compression of the heart (especially of the
right ventricle due to its thinner wall) → interventricular septum shift toward the
left ventricle chamber → ↓ systemic venous return (preload) → ↓ ventricular
diastolic filling → ↓ stroke volume (and venous congestion) → ↓ cardiac output
and equal end-diastolic pressures in all 4 chambers

362. Patient underwent lap chole for multiple gallstones 7 days ago,
presents with vague abdominal pain. On US there is fluid around the
gallbladder (something like that) and the CBD is 9 mm.
What is the most likely diagnosis or cause?
A - CBD injury
B - Retained stone in CBD
C - Sub-hepatic collection

• Correct Answer: B
TB : Caseating granuloma / Treatment: AKT
Sarcoidosis : Non Caseating Granuloma/ Treatment: Steroids
* A K T 4 Kit Tablet belongs to the group of medicines called anti-
tuberculosis drugs primarily used to treat and prevent tuberculosis. A K T 4
Kit Tablet is a combination of four anti-tuberculosis drugs, namely:
Rifampicin, isoniazid, pyrazinamide, and ethambutol.

363. Case about baby ingest aspirin large dose!! Toxicity Q what
abnormality in acid base balance? Fast breathing
Explanation:
Early: Respiratory alkalosis.
Late: Metabolic acidosis .

165
364. Baby underwent forceps delivery and the physician noticed crushing
the left sternomastoid foramen what the baby expected to have ?
A) Left side cant close his eye
B) Lost of sensation in the anterior 2/3 of the tongue

• Correct Answer:A
Explanation: Facial nerve pass through stylomastoid foramen so injury to the
foramen will result in an injury to the nerve resulting in the clinical features of
facial nerve palsy , because the injury is past the facial nucleus in the pons, this is a
lower motor lesion which will result in ipsilateral manifestation

365. Parents discover tonic clonic abnormal seizure of their child during
sleep
What is the Dd?
A) Nocturnal seizure
Need more details, this maybe refer to Benign (childhood) epilepsy with
centrotemporal spikes (BECTS) which occur during sleep
UpToDate:
Sleep-related epilepsy refers to syndromes in which seizures occur exclusively or
predominantly during sleep or in the period shortly after arousal from sleep.
Across the lifespan, these syndromes account for about 10 to 15 percent of all
epilepsy syndromes.
Sleep-related focal epilepsies have traditionally been named according to the
localization of seizure onset. The most common of these is sleep-related
hypermotor epilepsy (SHE), previously called nocturnal frontal lobe epilepsy

366. 47-year-old female presents with iron deficiency anemia. She has 3rd
degree hemorrhoids. Which of the following is the most appropriate thing
to do?
A - Colonoscopy
B - Hemorrhoidectomy
C - CT scan

166
• Correct Answer:A
Explanation: Any older pt with IDA should undergo colonscopy to rule out
malignancy
UpToDate: In addition, unexplained iron deficiency anemia should be evaluated
with colonoscopy, as colon cancer is an important cause of iron deficiency anemia
in adults.

367. case of female auditory hallucination And delusional For 1 month after
that she recover without treatment?
A) Brief psychotic episode.

• Correct Answer: A
AMBOSS:

UpToDate:
Brief psychotic disorder is defined in DSM-5 as the presence of one or more
psychotic symptoms with a sudden onset lasting more than one day and with full
remission within one month

368. Pt known HTN and DM found out she's pregnant what is your
management?
A) folic acid supplements.
B) ANC labs.
C) US.

167
D) review her medications

• Correct Answer:D
Explanation: Reviewing HTN and DM is essential to determine if she is using
a teratogenic medications for her HTN and DM and discontinue them or
switching to more appropriate medication for her pregnancy
UpToDate: Medication changes or discontinuation to avoid use of
teratogens

369. Pt vith dysmenorrhea what is the important to ask?


A) Family hx
B) Menstrual Hx
C) Medical hx
D) Surgical hx

• Correct Answer:B
Explanation:
The first thing to ask in the evaluation of pt with dysmenorrhea is menstrual
history:
UpToDate:
The evaluation of an adolescent female presenting with menstrual cramps begins
with a complete medical and menstrual history to assess the severity of the
symptoms and exclude secondary causes of dysmenorrhea
History — A complete history should include the following information (table 2A):
●Menstrual history:
•Age at menarche
•Duration of menstrual bleeding
•Menstrual flow assessment
•Interval between menstrual periods (from first day of one period to the
first day of the following period)
•First day of last two menstrual period

370. patient presenting with severe bleeding in 9th week of pregnancy, Os


is open, doctor saw some tissue on the cervix. What is your management?

168
A-Expectant management
B-D&C
C-Oxytocin

• Correct Answer:B
Explanation: This is a case of inevitable abortion, she is having a severe bleeding so
this require urgent intervention so A is wrong
AMBOSS:
Expectant management (option for women < 14 weeks gestation)
Medical evacuation: combination of mifepristone and misoprostol
Surgical evacuation (D&C): if spontaneous evacuation does not occur after 4 weeks
or in cases of septic abortion or heavy bleeding

371. Old patient hx of HF with dialated cardiomyopathy and A.fib . what is


the most appropriate management to control his heart rate?
A.propranolol
B. diltiazem/ verapamil
C. Digoxin

• Correct Answer:C
Explanation: need more details regarding his HF but will assume its
decompensated and choose C
UpToDate:
Initial therapy – In patients without significant heart failure or hypotension, we
suggest intravenous nondihydropyridine calcium channel blockers or beta blockers

In patients with advanced heart failure or significant hypotension, we suggest


digoxin as initial therapy
AMBOSS:
Pharmacological options for rate control
First-line:
Beta blockers (e.g., metoprolol, atenolol, propranolol)
Preferred when Afib is due to hyperthyroidism and in pregnant patients
Avoid in patients with COPD.
Nondihydropyridine calcium channel blockers (e.g., diltiazem, verapamil)

169
Avoid in patients with decompensated heart failure (LV systolic dysfunction/low
ejection fraction).
Can be safely used in heart failure with preserved normal LV systolic function.
Second line: digoxin; preferred as first-line therapy in patients with
decompensated HF when beta blockers are contraindicated.

372. Depressed female. What is the most important risk factor of suicidal
thoughts?
A- Age
B- Gender
C- Social isolation
D- Previous attempt.

• Correct Answer:D
Explanation: Not sure if they are asking for female specific risk factors or in
general, but will assume its risk factors in generals
AMBOSS:
Previous suicide attempt (most important risk factor)
UpToDate:
prior history of attempted suicide is the strongest single factor predictive of suicide

373. Highest risk factors for suicide?


A- Absence of medical illness
B- Social isolation
C- Age
D- Female

• Correct Answer:C vs B
AMBOSS:
Risk factors:
Age > 45 years
Male

170
374. A postmenopausal female complaining of vaginal itching and irritation.
There is vaginal discharge which she described as watery and odorless. On
examination, there were multiple scratches, and it had a scaly appearance,
and it bleeds with touch. What is your diagnosis?
A) Trichomoniasis
B) Atrophic vaginitis
C) Candida

• Correct Answer:B
Explanation:
A -> Greenish or yellow discharge + Foul smelling
B -> Postmenopausal + itching + Bleeds
C -> White + odorlesss

171
UpToDate: Atropic vaginitis:

375. Pt in ICU received 15 units of blood, now blood coming out from NGT,
incision, and cannula site
A) Transfusion reaction
B) Thrombocytopenia
C) Hypocalcemia
D) Von Willebrand

• Correct Answer:B
UpToDate:
Massive transfusion – Massive transfusion was historically defined as transfusion of
≥10 units of whole blood (WB) or red blood cells (RBCs) in 24 hours.

Gradual dilution of clotting proteins leads to prolongation of the prothrombin time


(PT) and activated partial thromboplastin time (aPTT). In an adult, there will be an

172
approximate 10 percent decrease in the concentration of clotting proteins for each
500 mL of blood loss that is replaced with plasma-poor pRBCs in additive solution.
Additional bleeding based solely on dilution can occur when the level of individual
coagulation proteins falls to <25 percent of normal. This usually requires 6 to 10
units of pRBCs in an adult.

A similar dilutional effect on the platelet concentration can be seen with massive
transfusion [31]. In an adult, each 10 to 12 units of transfused pRBCs are
associated with a 50 percent fall in the platelet count, and significant
thrombocytopenia can be seen after 10 to 20 units of blood, with platelet counts
<50,000/microL.

376. 16 yo male CO 3 years altered bowel habit he has 3-4x a day diarrhea
that is foul
smelling containing undigested food, no blood or mucus. He has post
prandial abdominal
distention (no mention of the type of the food). No secondary sexual
characteristics
(High LFT)
Most appropriate investigation?
A. Stool fat test
B. Barium follow though
C. Lactose Hydrogen breath test
D. IgA endomysia antibody

• Correct Answer:D
Explanation: He has malabsorption symptoms + growth delay which goes more
with celiac than lactose

UpToDate:
Serologic tests for celiac disease are useful for screening and are an important first
step in the diagnosis of the disease.
Anti-endomysial antibodies – Testing for anti-endomysial antibodies (EMA) is as
accurate as tTG-IgA, but this test is more expensive and somewhat dependent on
operator interpretation.

173
377. Pt of stab wound 2 cm penetrating injury with omentum passing out
Pt is stable Ct report negative findings, Next step management
A) observation
B) close the wound
C) leave the wound open
D) Laparotomy

• Correct Answer:D
UpToDate:
Evisceration of intra-abdominal organs or omentum is an indication for immediate
laparotomy at most trauma centers.

378. pt DM- HTn cardiac dis, long scenario. And all treatment given and
stable on thrombolytic, what is the most common cause of death during his
hospital stay:
A. bleeding
B. stroke
C. PE
D.MI

• Correct Answer: Not sure

379. A 35 year old lady presents with a left nipple bloody discharge, by
imaging it was suggestive of Intraductal papilloma. What to do next?
A- Central Duct excision
B- Observation
C- Mastectomy
D. image guided biopsy

• Correct Answer:D
UpToDate:

174
When a core biopsy demonstrates papilloma with atypical cells, surgical excision is
warranted

175
AMBOSS:

380. What is the rational of antihypertensive medication in preeclampsia?


A) Decrease UGR
B) Decrease mothers’ mortality
C) Decrease fetus mortality

• Correct Answer:B
most likely as it decrease the risk of stroke in the mother

381. 56 YO female presented with a 2 months history of jaundice associated


with lethargy & polyarthralgia. On examination, she is jaundiced & has
clubbing. There are several spider naevi on the front & back of the trunk.
Her abdomen is soft & there is a smooth hepatomegaly. Prior to her onset
of symptoms, the patient has been fit & well. Her LFTS reveal a bilirubin of
46 IU/L, AST 200, ALT 175, ALP 104. Viral serology is -ve & anti-soluble liver
antigen is detected. You decide to start this patient on management. What
is the most appropriate management?
A. Liver transplantation
B. Methotrexate
C. Prednisolone
D. CyclospoAntivir

• Correct Answer:C

176
Explanation:
Autoimmune hepatitis has bimodal distribution 10-20 and 40-60 years
Present in females more than males, and come with symptoms and signs of liver
failure. Anti-SLA is positive in this pt which is suggestive of Autoimmune hepatitis,
along with negative viral serology which rule out viral cause for this pt.

UpToDate:
Initial treatment for autoimmune hepatitis is typically with a glucocorticoid, with or
without azathioprine or 6-mercaptopurin
AMBOSS:
Immunosuppressive medications
Induction therapy
First-line: prednisone with or without azathioprine
Second-line: mycophenolate, cyclosporine, tacrolimus
Maintenance therapy: low doses of azathioprine or prednisone

382. Patient after 2 days post mi complains of chest pain radiate to back
with st segment elevation in leads II, III, and AVF present now with a new
holosystolic murmur and bilateral basal crackles. Diagnosis??
A- Right ventricular infarction
B- papillary muscle rupture
C-VSD
D-cardiac tamponade

• Correct Answer:B

UpToDate:
The clinical manifestations of hemodynamically significant papillary muscle rupture
include the acute onset of hypotension and severe pulmonary edema. On physical
examination, the precordium may be hyperactive, and systolic murmur may be
present. Typically a mid-, late-, or holosystolic murmur is present that may have
widespread radiation. Although the murmur may be loud, a thrill is generally not
present. Furthermore, many patients have no or only a soft murmur. In addition,
pulmonary edema and mechanical ventilation may mask the murmur.

177
383. Postnatal which should b treated first?
A-autism
B-dip in sacral area
C-tof
D-undescended testis

• Correct Answer:C
Sacral dimple don’t require treatment
Undescended testis doesn’t require treatment before 1 year of age
TOF is more urgent to repair because 50% of pts don’t survive without surgery

UpToDate:
Most patients with TOF undergo complete repair as their initial intervention by one
year of age (typically before six months of age)

Surgical treatment of congenitally undescended testes is recommended as soon as


possible after four months of age and definitely should be completed before the
child is two years old

384. GA 39 weeks having prom of 24 hours..ctg normal and no contraction


not in labor what to do??
A-c/s
B-iol

178
C-observation
D-augmentation of labor

• Correct Answer:B
Dr. Wafaa:
This is a case of PROM: Rupture of membranes before the onset of labor at term
(≥37 weeks)
Investigations:
⁃ Speculum Examination: Pooling of amniotic fluid in the posterior vaginal
vault observed is the gold standard
Management:
• Induction of labor is recommended.
• Expectant management for up to 12–24 hours is reasonable in otherwise
uncomplicated pregnancy and in the absence of infection.
• GBS Prophylaxis is indicated in case of rupture of membrane for 18hrs
and more or fever 38C!

385. Pt with hepatitis C and cirrhosis came to the ER drowsy with abdomen
distention, on exam abdomen is tense with lower bilateral limb edema,
what is the management?
1- perform paracentesis
2- start diuretic and spironolactone
3- TIPS

• Correct Answer:A
UpToDate:
Abdominal paracentesis is central to determining the cause of ascites and in ruling
out or confirming spontaneous bacterial peritonitis (SBP). In patients with SBP,
mortality increases by 3.3 percent/hour of delay in performing a paracentesis
Indications — There are several generally accepted indications for abdominal
paracentesis (table 1):

●Evaluation of new onset ascites.

179
●Testing of ascitic fluid in a patient with preexisting ascites who is admitted
to the hospital, regardless of the reason for admission.

●Evaluation of a patient with ascites who has signs of clinical deterioration,


such as fever, abdominal pain/tenderness, hepatic encephalopathy,
peripheral leukocytosis, deterioration in renal function, or metabolic
acidosis.
AMBOSS:
Indications
-All patients with new-onset ascites (to identify the underlying etiology)
-To detect spontaneous bacterial peritonitis (SBP) or other peritoneal
infections in the following situations:

386. 48 Y/O very obese lady,BMI 41 had plevic organ prolapse many time
and then had surgery, what do you expect she’s having now :
A- Entrocele
B- Rectcele
C- Uterine prolapse
D-Cytocele
E-Vesicocele

• Correct Answer:A

387. A pregnant lady complains of vaginal bleeding, she a confirmed case of


low lying placenta, no active bleeding now and no pain. Next step in
management?
A- Ultrasound
B- Biophysical profile
C- CTG
D- Emergency delivery

• Correct Answer:C vs A

180
Dr. Wafaa: The first thing to do in antepartum hemorrhage is US to exclude
placenta previa. Then CTG.
But she is already diagnosed with placenta previa, so I would go with CTG

AMBOSS:
If low-lying placenta or placenta previa without placenta accreta is diagnosed:
repeat transvaginal ultrasound at 36 weeks gestation.

388. Patient with hx of pancreatitis months ago and now referred to surgery
clinic for mild abdominal pain and , what initial inves?
A-U/S
B- CT
C- endoscope

• Correct Answer:A
This depends on if this is chronic pancreatisis or pseudocyst
If picture of pseudocyst: US
If chronic pancreatitis: CT
UpToDate:
Pancreatic pseudocysts may be seen on transabdominal ultrasound, but contrast-
enhanced CT or MRI is typically obtained to confirm the diagnosis and to further
classify the fluid collection.

Another recall:
389. Patient with hx of pancreatitis month ago and now referred to surgery
clinic for mild abdominal pain and , what initial inves? I think it was
psudocyst
A-U/S
B- Barium
C- endoscopy
I got this Question and I’m sure there’s no CT in options

• Correct Answer:A

181
AMBOSS:
Transabdominal ultrasound (pseudocyst)
Fast and readily available
High sensitivity, but low negative predictive value

390. Questions about Milestones:


Dress himself-- Answer:5 years (Nelson 4 years)
Play with group-- Answer:5 years
Tie shoe-- Answer:6 years

391. eldery smoker k/c of poorly controlled DM comes with ulcers on tip of
three of his toes, diminished dorsalis pedis bilaterally, however, intact
popliteal pulse, what’s the initial management;
A- Amputation
B- Long term anticoagulation
C- Immediate surgical intervention
D- Diet modification and lifestyle changes

Correct Answer: C
UpToDate: Severe ischemia is manifest on clinical examination as severely
diminished or absent pedal pulses, dependent rubor, ABI <0.40, or toe pressure
<30 mmHg. Vascular imaging is warranted for those with obvious limb ischemia.
Tissue loss can manifest as ischemic ulceration or as wet or dry gangrene. Patients
with stable, dry gangrene do not require immediate debridement; vascular imaging
and revascularization are preferentially performed before debridement.
Schwartz: These wounds commonly are present at the most distal portions of the
extremities such as the interdigital clefts, although more proximal locations are
also encountered. On examination, there may be diminished or absent pulses with
decreased ankle-brachial index and poor formation of granula- tion tissue. Other
signs of peripheral ischemia, such as dryness of skin, hair loss, scaling, and pallor
can be present. The wound itself usually is shallow with smooth margins, and a
pale base and surrounding skin may be present. The management of these wounds
is two-pronged and includes revascularization and wound care. Nonhealing of
these wounds is the norm unless successful revascularization is performed. After

182
establishing adequate blood supply, most such wounds progress to heal
satisfactorily.

392. A 2-month baby came with SOB and centroperipheral cyanosis she was
diagnosis after birth but his parents forgot diagnosis what's likely diagnosis
A -TOF
B- ASD
C- VSD
D- coarctation of aorta

• Correct Answer:A
Explanation:
ASD, VSD are both fall under Acynotic congenital heart failure
Coarcation of aorta present with specific type of cyanosis:
Differential cyanosis: cyanosis of the lower extremities (when the left
subclavian artery outflow is involved, cyanosis might also be seen in the left
arm) also different blood pressure finding

393. Increase cardiac out put , increase... , decrease ... resistance what type
of shock
A- cardiogenic
B- Neurogenic
C- Septic
D- Hypovolemic

• Correct Answer: Not enough info probably C


Explanation: Septic shock: Early: compensatory ↑ HR and contractility → ↑ CO
(hyperdynamic state or “warm shock”)

394. A case of AAA in a hemodynamic unstable patient. They painted the


scenario that the man ate food and after a while he started having severe
abdominal pain despising giving analgesic. He became confused and

183
unconscious later in the hospital. PE: Tender and pulsatile mass, BP low.
What is your most appropriate mgt ?
A. US
B. CT
C. Ex Lap

• Correct Answer:C
Explanation: Unstable pt need urgent surgical repair for AAA
AMBOSS:
Invasive treatment: AAA repair
Indications [1]
Emergency repair: unstable patients

395. Whiff’s test positive. What is your diagnosis


A. Candidiasis
B. Trichomoniasis
C. Bacterial vaginosis

• Correct Answer: C

396. Patient on oxytocin, epidural and MgS04, preeclampsia. Her CTG:


absence variability (or non-reactive). What's the cause?
A. MgS04 toxicity
B. Oxytocin
C. Epidural analgesia

• Correct Answer:A
Dr. Wafaa: A

184
397. child have dry non purulent conjunctivitis, cracked red lips, erythema (I
think trunk, sole, hand) what’s the diagnosis?
A) Kawasaki disease
B) Rubella
C) Measles

• Correct Answer:A
AMBOSS:
Conjunctivitis, Rash, Adenopathy, Strawberry tongue, Hands and feet, and BURN
(fever ≥ 5 days) are the most common features of Kawasaki disease.

398. Child came with anal itching mother noticed rice like, whats the
organism responsible:
A-enterobius vermicularis

AMBOSS:
Clinical features:
Anal pruritus (especially at night)
Vulvovaginitis, especially in children
Occasionally, symptoms of intestinal infection (i.e., nausea, vomiting, and
abdominal pain which may become severe enough to mimic appendicitis)

399. Patient came with right-sided pleural effusion and a positive history of
hemoptysis. There was no shift of the trachea nor the cardiac apex. What is
the diagnosis?
A. Rupture of the esophagus
B. Heart failure
C. Tuberculosis
D. Cancer obstructing the ipsilateral bronchus

• Correct Answer: C Need more details but hemoptysis is one of the cardinal
symptoms of TB infection and can present with exudative pleural effusion.

185
More details:
“This question came in my exam

Here is some Extra details:


Patient had chronic cough and weight loss, Pleural effusion analysis was
positive for AFB
Answer is correct as AFB is specific for TB”

400. World health organization (WHO) to determine the health of the


population in KSA What should KSA share to WHO:
A) Health determinant
B) Health indicator
C) Risk factors
D) Something variable

• Correct Answer:B
Explanation: Health indicators are measures designed to summarize
information about priority topics related to population health or health
system performance.
CDC: Health indicators :A measurable characteristic that describes: – the
health of a population.
Social determinants of health: are the non-medical factors that influence
health outcomes.

401. Child with recurrent Sinopulmonary infection ard Gi infection His


brother dead 6 morth due to sepsis What is the most likely diagrosis?
A Iga deficiency
B) X-linked chronic granulomatous disease

• Correct Answer: B
AMBOSS:
X-linked chronic granulomatous disease is x-linked recessive disease, so this explain
his brother death + Symptoms present 3 to 6 months of age
The frequent sites of infection are lung, skin, lymph nodes, and liver.

186
402. Hx of psychological disorders?
A.past Suicidal attempts
A.Social isolation
B.Use of alcohol or drugs

• Correct Answer: A
UpToDate: a prior history of attempted suicide is the strongest single factor
predictive of suicide.

Another recall:
Highest risk factors for suicide?
A-Absence of medical illness
B- Social isolation
C- Age
D- Female

• Correct Answer: B
UpToDate: The risk of suicide increases in patients who live alone, have lost a
loved one, or have experienced a failed relationship within one year; and possibly
in patients with a history of violent behavior in the previous year.

187

You might also like